Physical Education (Theory) Official 2022-2023 ISC (Commerce) Class 12 Question Paper Solution

Advertisements.

  • Candidates are allowed additional 15 minutes for only reading the paper.
  • They must NOT start writing during this time.
  • Answer all questions in Section A and Section B.
  • Answer all questions on any two games in Section C.
  • The intended marks for questions or parts of questions are given in brackets [].

SAI was established in the year ______.

______ is a chemical substance that stimulates or excites a person mentally or physically.

The total number of matches in a knock-out tournament comprising thirty-four teams would be ______.

The competitions that are held within the school premises and the participants are also from the same school, are called ______.

______ are fluid filled bumps that look like bubbles on the skin and are caused due to friction from wearing ill-titted shoes.

Netaji Subhas National Institute of Sports was founded in ______.

Madras, 1963

Thiruvananthapuram, 1962

Calcutta, 1961

Patiala, 1961

Which one of the following is a soft tissue injury?

Joint injury

Bone injury

Select the correct sequence of the procedure for the treatment of soft tissue injuries.

Rest, Compression, Elevation, Ice

Compression, Elevation, Ice, Rest

Ice, Elevation, Compression, Rest

Rest, Ice, Compression, Elevation

A condition in which the entire sole of the foot touches the ground while standing is known as ______.

runner's knee

A type of exercise in which the muscles contract and change their length to produce force is known as ______.

Isometric exercise

Isokinetic exercise

Isotonic exercise

Warming up exercise

What is the full form of IOA?

Give the meaning of the term personal hygiene.

What type of deformity is Scoliosis?

Give the meaning of the term grazes.

What is weight training?

Briefly explain interval training.

Give any three advantages of interval training.

Write a short note on Narcotic Analgesics.

Write a short note on Warming-up exercises.

Write any two merits of the League Tournament.

List any three measures to prevent Air Pollution.

Draw a knock-out fixture consisting of eleven teams.

What are conditioning exercises?

Write any four advantages of the Conditioning exercise.

State the effects of insufficient sleep on the human body.

Explain the Fartlek Training method.

Explain the centre circle.

If a player deliberately kicks an opponent player during a match, what will be the decision of the referee?

During a match, the ball hits the referee. What will be the decision of the referee?

With the help of a diagram, show any two formations of players in the game of football.

State any five duties of a Football Coach.

Explain the following:

Write any three situations when an innings can be closed.

A fielder deliberately distracts a batsman by making verbal comments during a match. What is this practice called?

What will be the decision of the umpire if a batsman does not cross the crease while taking the second run?

Write a short note on Follow-on.

State any five decisions that can be given by Leg-umpire.

Explain the following term:

Explain the following term in cricket:

Sight screen

Explain the Tie-breaking process in the game of Hockey.

What will be the referee's decision when a player other than the goalkeeper touches the ball with their hands?

What will be the decision of the umpire in Field hockey if the ball touches the defender and goes out of the back line?

Draw a neat diagram of a Hockey field with all its dimensions.

State any five situations when an opponent player can get a free hit.

Duration of a Hockey match.

List any three jump ball situations in basketball.

If the ball goes out of the sideline, what will be the decision of the referee?

During a match, a player holds the ball while dribbling and runs. What will be the decision of the referee?

Explain time-out.

Explain the Key in the Basketball court.

Technical foul

Explain the term Free throw in basketball.

Explain any three types of zones in Volleyball.

While serving the ball, a player steps on the end line. What will be the referee's decision?

What will be the decision of the referee if the ball touches the net and falls into the opponent's court during service?

Write any five faults committed by the player while smashing the ball.

State any five duties of the Second Referee.

Technical time-out

Explain the order of service in Tennis.

Which player would gain a point if the ball touches the permanent fixture before hitting the ground?

During a rally, the ball touches the net and falls in the opponent's court. What is this considered as?

State any five conditions when a service is declared a fault by the umpire.

What is Grand Slam in tennis?

Name the different grand slam tournaments in tennis.

Explain the following term.

Eastern grip

Unforced error

Explain any three scoring systems in badminton.

What will be the decision of the umpire if the shuttlecock hits the ceiling during a match?

A shuttlecock lands on the line of the court during a rally. What will be the decision of the umpire?

State the rules of service in a game of doubles.

Draw a well labelled diagram of a badminton racket with all its specifications.

Explain the following term in Badminton.

Back Gallery

What is the procedure to start a Backstroke race?

What will be the decision of the referee if a player is absent from the event?

What will be the decision of the officials if a swimmer makes a false start?

Mention any five responsibilities of the control room supervisor in a tournament.

List any five freestyle events in swimming.

Dolphin Kick

Role of a Life Guard

State any three basic skills required by an athlete for the discus throw.

In a relay race, an athlete finishes the race, without holding a baton. What will be the referee's decision?

What will be the decision of the officials if the tail of the javelin lands on the ground after the throw?

Explain the different types of starts in Sprint races.

List any five Track events and any five Field events in Athletics.

Relay baton

Race Walking

Question Papers VIEW ALL [7]

  • view Question Papers For All Subjects
  • English Language 2011 to 2025 question paper
  • Hindi (Indian Languages) 2013 to 2024 question paper
  • Commerce 2014 to 2025 question paper
  • Accounts 2011 to 2025 question paper
  • Business Studies 2014 to 2025 question paper
  • Mathematics 2011 to 2025 question paper
  • Physical Education (Theory) 2013 to 2025 question paper

Video Tutorials VIEW ALL [1]

  • view Video Tutorials For All Subjects
  • Health video tutorial 00:24:43

Submit Question Paper

Cisce previous year question papers class 12 physical education (theory) with solutions 2022 - 2023, select sets to download.

Download the Shaalaa app from the Google Play Store

  • Maharashtra Board Question Bank with Solutions (Official)
  • Balbharati Solutions (Maharashtra)
  • Samacheer Kalvi Solutions (Tamil Nadu)
  • NCERT Solutions
  • RD Sharma Solutions
  • RD Sharma Class 10 Solutions
  • RD Sharma Class 9 Solutions
  • Lakhmir Singh Solutions
  • TS Grewal Solutions
  • ICSE Class 10 Solutions
  • Selina ICSE Concise Solutions
  • Frank ICSE Solutions
  • ML Aggarwal Solutions
  • NCERT Solutions for Class 12 Maths
  • NCERT Solutions for Class 12 Physics
  • NCERT Solutions for Class 12 Chemistry
  • NCERT Solutions for Class 12 Biology
  • NCERT Solutions for Class 11 Maths
  • NCERT Solutions for Class 11 Physics
  • NCERT Solutions for Class 11 Chemistry
  • NCERT Solutions for Class 11 Biology
  • NCERT Solutions for Class 10 Maths
  • NCERT Solutions for Class 10 Science
  • NCERT Solutions for Class 9 Maths
  • NCERT Solutions for Class 9 Science
  • CBSE Study Material
  • Maharashtra State Board Study Material
  • Tamil Nadu State Board Study Material
  • CISCE ICSE / ISC Study Material
  • Mumbai University Engineering Study Material
  • CBSE Previous Year Question Paper With Solution for Class 12 Arts
  • CBSE Previous Year Question Paper With Solution for Class 12 Commerce
  • CBSE Previous Year Question Paper With Solution for Class 12 Science
  • CBSE Previous Year Question Paper With Solution for Class 10
  • Maharashtra State Board Previous Year Question Paper With Solution for Class 12 Arts
  • Maharashtra State Board Previous Year Question Paper With Solution for Class 12 Commerce
  • Maharashtra State Board Previous Year Question Paper With Solution for Class 12 Science
  • Maharashtra State Board Previous Year Question Paper With Solution for Class 10
  • CISCE ICSE / ISC Board Previous Year Question Paper With Solution for Class 12 Arts
  • CISCE ICSE / ISC Board Previous Year Question Paper With Solution for Class 12 Commerce
  • CISCE ICSE / ISC Board Previous Year Question Paper With Solution for Class 12 Science
  • CISCE ICSE / ISC Board Previous Year Question Paper With Solution for Class 10
  • Entrance Exams
  • Video Tutorials
  • Question Papers
  • Question Bank Solutions
  • Question Search (beta)
  • More Quick Links
  • Privacy Policy
  • Terms and Conditions
  • Shaalaa App
  • Ad-free Subscriptions

Select a course

  • Class 1 - 4
  • Class 5 - 8
  • Class 9 - 10
  • Class 11 - 12
  • Search by Text or Image
  • Textbook Solutions
  • Study Material
  • Remove All Ads
  • Change mode
  • Sample Paper
  • Question Paper
  • NCERT Solutions
  • NCERT Books
  • NCERT Audio Books
  • NCERT Exempler
  • Model Papers
  • Past Year Question Paper
  • Writing Skill Format
  • RD Sharma Solutions
  • HC Verma Solutions
  • CG Board Solutions
  • UP Board Solutions
  • Careers Opportunities
  • Courses & Career
  • Courses after 12th

Home » 12th Class » ISC Class 12 Physical Education Question Paper 2023 (PDF) – Class 12 Physical Education Previous Year Question Papers

ISC Class 12 Physical Education Question Paper 2023 (PDF) – Class 12 Physical Education Previous Year Question Papers

ISC Class 12 Physical Education Question Paper 2023 is available for download here. You can see all the questions asked in previous year CISCE Class 12th Physical Education exam from here on aglasem and practice them to excel in your test. The ISC Class 12 previous year question paper for Physical Education contain exact questions framed by the Council for the Indian School Certificate Examinations in that session. Therefore by solving the Physical Education paper, you can boost your own exam preparation for upcoming board exams.

ISC Class 12 Physical Education Question Paper 2023

The CISCE previous year question paper of ISC Class 12 for Physical Education are as follows.

ISC Class 12 Physical Education Question Paper 2023 PDF Download Link – Click Here to Download Question Paper

ISC Class 12 Physical Education Question Paper 2020 PDF Download Link – Click Here to Download Question Paper

ISC Class 12 Physical Education Question Paper 2019 PDF Download Link – Click Here to Download Question Paper

ISC Class 12 Physical Education Question Paper 2023 PDF

The complete Physical Education question paper of last year is as follows. However you may find more ISC question paper pdf download link above itself.

isc physical education solved question paper

ISC Class 12 Previous Year Question Papers

There are multiple subjects in class 12. To do well in CISCE class 12 exams, you should solve past year papers for all. So here are all previous year question paper PDFs of ISC class 12.

  • Art Paper 1 – Drawing Or Painting From Still Life
  • Art Paper 2 – Drawing Or Painting From Nature
  • Art Paper 3 – Drawing Or Painting Of A Living Person
  • Art Paper 4 – Original Imaginative Composition In Colour
  • Art Paper 5 – Crafts
  • Biotechnology
  • Business Studies
  • Computer Science
  • Elective English
  • Electricity and Electronics
  • English Language
  • English Literature
  • Environmental Science
  • Fashion Designing
  • Geometrical & Building Drawing
  • Geometrical & Mechanical Drawing
  • Home Science
  • Hospitality Management
  • Indian Music Carnatic
  • Indian Music Hindustani
  • Legal Studies
  • Mass Media & Communication
  • Mathematics
  • Modern Armenian
  • Physical Education
  • Political Science
  • Western Music

CISCE Previous Year Question Papers

You can explore all past year papers of CISCE board here on aglasem.

ISC Class 12 Physical Education Previous Year Question Paper – An Overview

The important features of this paper set are as follows.

AspectsDetails
Board
ClassClass 12
SubjectPhysical Education
Resources HereCISCE Previous Year Question Papers of Class 12 Physical Education
All Question Papers of 2023
More PYQP of This Class
All PYQP of This Board
Sample Papers
Date Sheet
Results

If you have any queries on ISC Class 12 Physical Education Question Paper 2023, then please ask in comments below.

AglaSem Earn while Learn Program. Send your papers and get paid. Contact: [email protected]

To get study material, exam alerts and news, join our Whatsapp Channel .

ISC Class 12 Geometrical & Building Drawing Question Paper 2023 (PDF) – Class 12 Geometrical & Building Drawing Previous Year Question Papers

Isc class 12 mass media & communication question paper 2023 (pdf) – class 12 mass media & communication previous year question papers, related posts.

Class 12 Exam Date

Goa HSSC Time Table 2025 (PDF Out) – Download GBSHSE Class 12th Exam Date

Tamil Nadu Board

Tamil Nadu Board Class 12 Previous Year Question Paper (PDF) – Download Tamil Nadu Board Question Papers

Tn 12th public exam telugu question paper 2024 pdf | download tamil nadu board pyqp, tn 12th public exam office management & secretaryship question paper 2024 pdf | download tamil nadu board pyqp, leave a reply cancel reply, cbse board quick links.

  • CBSE Date Sheet
  • CBSE Result
  • CBSE Syllabus
  • CBSE Sample Papers
  • CBSE Question Papers
  • CBSE Practice Papers

CISCE Board Quick Links

  • CISCE Time Table
  • CISCE Results
  • CISCE Specimen Papers
  • CISCE Syllabus
  • CISCE Question Papers

Class Wise Study Material

Board exams 2023.

  • Solved Sample Papers
  • Revision Notes
  • State Board

Study Material

  • Class Notes
  • Courses After Class 12th
  • JEE Main 2024
  • Fashion & Design
  • Terms of Use
  • Privacy Policy

© 2019 aglasem.com

Discover more from AglaSem Schools

Subscribe now to keep reading and get access to the full archive.

Continue reading

CollegeDekho

Frequently Search

Couldn’t find the answer? Post your query here

  • CISCE Board
  • CISCE Board 12th
  • ISC Class 12 Physical Education Previous Year Question Paper

ISC Class 12 Physical Education Previous Year Question Paper: Download PDF

Updated On: June 10, 2024 01:10 am IST

  • ISC Class 12 Physical Education Previous Year Question Paper: Download …
  • How to Download ISC Class 12 Physical Education Previous Year …
  • Benefits of Solving ISC Class 12 Physical Education Previous Year …

ISC Class 12 Physical Education Previous Year Question Paper

Never Miss an Exam Update

ISC Class 12 Physical Education Previous Year Question Paper: The Council for the Indian School Certificate Examinations (CISCE) publishes the ISC Class 12 Physical Education Previous Year Question Papers at cisce.org. The ISC 12th Physical Education is divided into two papers, Paper I (Theory) and Paper II (Project work). Paper I of 70 marks is further divided into two sections, Sections A and B. In Section A, students will have to answer 5 out of 7 questions, each carrying 8 marks. Section B will comprise questions on major games. Candidates will have to choose two games from this section answering any 3 out of 5 subparts from each of the two selected games they have chosen. Here, each question will have 15 marks

The ISC class 12 Physical Education practical work will be held for 30 marks. The council will not set any question paper for the practical exam. The ISC 12th Physical Education practical work will be evaluated based on Continuous Evaluation (by the Teacher) which will carry 10 marks, and Practical Evaluation (by Visiting Examiner) which will carry 20 marks. There will be a total of 3 hours to complete the ISC 12th Physical Education Theory Exam 2025. The students will be given an extra 15 minutes to read the question paper. CISCE will conduct the ISC class 12 exam 2025 from February to March 2025 generally in two shifts. Here, we have provided the ISC Class 12 Physical Education Previous Year Question Paper in this article.

ISC Class 12 Physical Education Previous Year Question Paper: Download PDFs

Students can check and download the previous year ISC Class 12 Physical Education Question Paper PDFs tabulated below:

ISC Class 12 Physical Education Question Paper 2023

ISC Class 12 Physical Education Question Paper 2020

ISC Class 12 Physical Education Question Paper 2019

ISC Class 12 Physical Education Question Paper 2018

ISC Class 12 Physical Education Question Paper 2017

How to Download ISC Class 12 Physical Education Previous Year Question Paper?

Follow the simple steps provided below to download the ISC 12th Physical Education Question Paper PDFs from the official website of CISCE:

  • Step 1: Students need to open the official website of CISCE - cisce.org
  • Step 2: On the homepage, click on “Examinations” from the navigation bar.
  • Step 3: Then click on the link “ISC Examination”.
  • Step 4: On the new window that will open, scroll down to the link “Previous Years Question Papers”.
  • Step 5: You will find the ISC 12th Question Papers of 5 different years.
  • Step 6: Click on the year you want to download.
  • Step 7: Download the ISC 12th Physical Education Previous Years Question Papers and start practicing.

Benefits of Solving ISC Class 12 Physical Education Previous Year Question Paper

The best way to get acquainted with the ISC 12th exam 2025 is by solving the ISC class 12 Physical Education previous year question papers. Students must know what are the benefits of practicing the previous year question papers:

  • Opting to solve the previous year question papers of ISC 12th Physical Education can help the students evaluate their strengths and weaknesses. As per that, students can work on the areas they are lagging.
  • Students will also be able to get an insight into the ISC 12th Physical Education exam pattern, marking scheme, typologies and difficulty level of the questions.
  • Also, they will get to know the important and repeated questions.
  • Solving the previous year ISC 12th Physical Education question papers will provide a sneak preview of the main exam.
  • Additionally, it ensures the development of effective time management skills. By practicing the previous year question papers students can calculate the time that will take each answer and hence the whole question paper.
  • Overall, solving the ISC 12th Physical Education previous year question papers is the best revision strategy for the ISC 12th Physical Education Exam.

Solving the ISC Class 12 Physical Education Previous Year question papers can be an excellent self-assessment tool that helps the students evaluate their progress for the ISC 12th examination. 

Students who are appearing for the upcoming ISC 12th exams in 2024 can access the previous year ISC 12th Physical Education question papers from this page. They can also access the question papers from the official website of CISCE.

You can get the important or repeated questions. However, the questions will not come in the same manner. The same questions will come in a different way.

Was this article helpful?

Do you have a question ask us..

Typical response between 24-48 hours

Get personalized response

Free of Cost

Access to community

  • ISC Class 12th Improvement Result 2024 (Out)- CISCE ISC 12th Compartment Result at cisce.org, Direct Link Here
  • ISC Class 12 Preparation Tips 2025
  • ISC Class 12th Syllabus 2024-25
  • ISC Class 12 Exam Pattern 2024-25
  • ISC 12th Board Exam 2025
  • ISC Class 12 Biology Previous Year Question Paper
  • ISC Class 12 Business Studies Previous Year Question Paper
  • ISC Class 12 Accountancy Previous Year Question Paper

Subscribe to CollegeDekho News

  • Select Stream Engineering Management Medical Commerce and Banking Information Technology Arts and Humanities Design Hotel Management Physical Education Science Media and Mass Communication Vocational Law Others Education Paramedical Agriculture Nursing Pharmacy Dental Performing Arts

Trending Articles

Up board 12th result 2024 released - up board class 12 result for arts, commerce and science streams at upmsp.edu.in, bihar board 12th result 2024 (released) - bseb class 12 result for arts, commerce and science streams at biharboardonline.bihar.gov.in, ap inter 2nd result 2024 (released) - bieap 2nd year result for commerce and science streams at bieap.apcfss.in, jac 12th result 2024 (out) - jharkhand board class 12th result direct link, toppers here, isc 12th result 2024 (released) - isc class 12th result date, marksheet, topper at results.cisce.org, tamil nadu 12th result 2024 (released) - check tn hsc result release date for all streams & how to check tn hsc result 2024 @tnresults.nic.in, maharashtra hsc result 2024 (released) - maharashtra board class 12 result @mahresult.nic.in for all streams, how to check result, cbse class 12 result 2024 (out) - cbse 12th result for arts, commerce and science streams at cbseresults.nic.in, rbse 12th result 2024 (released) - rajasthan board class 12th result at rajeduboard.rajasthan.gov.in for all streams, karnataka 2nd puc result 2024 (released)- check karnataka class 12 result at karresults.nic.in., mp board 12th result 2024 declared @mpresults.nic.in - check mpbse class 12 result date & check here for all streams.

CollegeDekho's expert counsellors can help you with all your doubts

  • Enter a Valid Name
  • Enter a Valid Mobile
  • Enter a Valid Email
  • By proceeding ahead you expressly agree to the CollegeDekho terms of use and privacy policy

Details Saved

isc physical education solved question paper

Your College Admissions journey has just begun !

Try our AI-powered College Finder. Feed in your preferences, let the AI match them against millions of data points & voila! you get what you are looking for, saving you hours of research & also earn rewards

For every question answered, you get a REWARD POINT that can be used as a DISCOUNT in your CAF fee. Isn’t that great?

1 Reward Point = 1 Rupee

Basis your Preference we have build your recommendation.

cd_logo

  • Study Abroad Get upto 50% discount on Visa Fees
  • Top Universities & Colleges
  • Abroad Exams
  • Top Courses
  • Read College Reviews
  • Admission Alerts 2024
  • Education Loan
  • Institute (Counselling, Coaching and More)
  • Ask a Question
  • College Predictor
  • Test Series
  • Practice Questions
  • Course Finder
  • Scholarship
  • All Courses
  • B.Sc (Nursing)

ISC logo

Collegedunia Team

Content Curator

ISC Class 12 Physical Education Paper 1 Question Paper 2023 with Answer Key pdf is available for download here. The exam was conducted by Council for the Indian School Certificate Examinations (CISCE) on March 24, 2023 in the Afternoon Session 2 PM to 5 PM. The question paper comprised a total of 14 questions divided among 3 sections.

ISC Class 12 Physical Education Paper 1 Question Paper 2023 with Answer Key

ISC Class 12 Physical Education Question Paper 2023 PDF ISC Class 12 Physical Education Answer Key 2023 PDF

isc physical education solved question paper

ISC Class 12 Previous Year Question Papers

ISC Practice Papers

ISC Question Paper

ISC Question Paper 2023

ISC Question Paper 2022

ISC Question Paper 2020

ISC Question Paper 2019

ISC Question Paper 2018

ISC Question Paper 2017

ISC Question Paper 2024

ISC Overview

ISC Registration

ISC Exam Pattern

ISC Preparation Tips

isc physical education solved question paper

ISC Improvement Result 2024 Declared

isc physical education solved question paper

isc class 12 Math question paper 2024

isc physical education solved question paper

ISC Class 12 Psychology Exam 2024 Postponed Due to Missing Question Paper New Date Announc...

isc physical education solved question paper

isc class 12 Economics question paper 2024...

Want to know more?

Ask questions to our experts

SUBSCRIBE TO OUR NEWS LETTER

downloadapp_banner image

Self Studies

  • Andhra Pradesh
  • Chhattisgarh
  • West Bengal
  • Madhya Pradesh
  • Maharashtra
  • Jammu & Kashmir
  • NCERT Books 2022-23
  • NCERT Solutions
  • NCERT Notes
  • NCERT Exemplar Books
  • NCERT Exemplar Solution
  • States UT Book
  • School Kits & Lab Manual
  • NCERT Books 2021-22
  • NCERT Books 2020-21
  • NCERT Book 2019-2020
  • NCERT Book 2015-2016
  • RD Sharma Solution
  • TS Grewal Solution
  • TR Jain Solution
  • Selina Solution
  • Frank Solution
  • Lakhmir Singh and Manjit Kaur Solution
  • I.E.Irodov solutions
  • ICSE - Goyal Brothers Park
  • ICSE - Dorothy M. Noronhe
  • Micheal Vaz Solution
  • S.S. Krotov Solution
  • Evergreen Science
  • KC Sinha Solution
  • ICSE - ISC Jayanti Sengupta, Oxford
  • ICSE Focus on History
  • ICSE GeoGraphy Voyage
  • ICSE Hindi Solution
  • ICSE Treasure Trove Solution
  • Thomas & Finney Solution
  • SL Loney Solution
  • SB Mathur Solution
  • P Bahadur Solution
  • Narendra Awasthi Solution
  • MS Chauhan Solution
  • LA Sena Solution
  • Integral Calculus Amit Agarwal Solution
  • IA Maron Solution
  • Hall & Knight Solution
  • Errorless Solution
  • Pradeep's KL Gogia Solution
  • OP Tandon Solutions
  • Sample Papers
  • Previous Year Question Paper
  • Important Question
  • Value Based Questions
  • CBSE Syllabus
  • CBSE MCQs PDF
  • Assertion & Reason
  • New Revision Notes
  • Revision Notes
  • Question Bank
  • Marks Wise Question
  • Toppers Answer Sheets
  • Exam Paper Aalysis
  • Concept Map
  • CBSE Text Book
  • Additional Practice Questions
  • Vocational Book
  • CBSE - Concept
  • KVS NCERT CBSE Worksheets
  • Formula Class Wise
  • Formula Chapter Wise
  • JEE Previous Year Paper
  • JEE Mock Test
  • JEE Crash Course
  • JEE Sample Papers
  • Important Info
  • SRM-JEEE Previous Year Paper
  • SRM-JEEE Mock Test
  • VITEEE Previous Year Paper
  • VITEEE Mock Test
  • BITSAT Previous Year Paper
  • BITSAT Mock Test
  • Manipal Previous Year Paper
  • Manipal Engineering Mock Test
  • AP EAMCET Previous Year Paper
  • AP EAMCET Mock Test
  • COMEDK Previous Year Paper
  • COMEDK Mock Test
  • GUJCET Previous Year Paper
  • GUJCET Mock Test
  • KCET Previous Year Paper
  • KCET Mock Test
  • KEAM Previous Year Paper
  • KEAM Mock Test
  • MHT CET Previous Year Paper
  • MHT CET Mock Test
  • TS EAMCET Previous Year Paper
  • TS EAMCET Mock Test
  • WBJEE Previous Year Paper
  • WBJEE Mock Test
  • AMU Previous Year Paper
  • AMU Mock Test
  • CUSAT Previous Year Paper
  • CUSAT Mock Test
  • AEEE Previous Year Paper
  • AEEE Mock Test
  • UPSEE Previous Year Paper
  • UPSEE Mock Test
  • CGPET Previous Year Paper
  • Crash Course
  • Previous Year Paper
  • NCERT Based Short Notes
  • NCERT Based Tests
  • NEET Sample Paper
  • Previous Year Papers
  • Quantitative Aptitude
  • Numerical Aptitude Data Interpretation
  • General Knowledge
  • Mathematics
  • Agriculture
  • Accountancy
  • Business Studies
  • Political science
  • Enviromental Studies
  • Mass Media Communication
  • Teaching Aptitude
  • Verbal Ability & Reading Comprehension
  • Logical Reasoning & Data Interpretation
  • CAT Mock Test
  • CAT Important Question
  • CAT Vocabulary
  • CAT English Grammar
  • MBA General Knowledge
  • CAT Mind Map
  • CAT Study Planner
  • CMAT Mock Test
  • SRCC GBO Mock Test
  • SRCC GBO PYQs
  • XAT Mock Test
  • SNAP Mock Test
  • IIFT Mock Test
  • MAT Mock Test
  • CUET PG Mock Test
  • CUET PG PYQs
  • MAH CET Mock Test
  • MAH CET PYQs
  • NAVODAYA VIDYALAYA
  • SAINIK SCHOOL (AISSEE)
  • Mechanical Engineering
  • Electrical Engineering
  • Electronics & Communication Engineering
  • Civil Engineering
  • Computer Science Engineering
  • CBSE Board News
  • Scholarship Olympiad
  • School Admissions
  • Entrance Exams
  • All Board Updates
  • Miscellaneous
  • State Wise Books
  • Engineering Exam

ISC Class 12 Physical Education Sample Paper 2023-24 Solved PDF

Free pdf download.

SHARING IS CARING If our Website helped you a little, then kindly spread our voice using Social Networks. Spread our word to your readers, friends, teachers, students & all those close ones who deserve to know what you know now.

The Council for the Indian School Certificate Examinations (CISCE) publishes the Class 12 Physical Education Specimen to understand what to expect in the exam. By solving the ISC Class 12 Physical Education Sample Paper 2023-24, you can understand the different types of questions: multiple choice questions, match the following, and subjective questions. Through this, you can learn to approach the same questions while attempting the Class 12 Physical Education exam. 

Physical Education Sample Paper Class 12 2023-24 ISC PDF

During the Class 12 Physical Education preparation, it is advisable for you to solve more questions. You can solve more questions from the Physical Education Sample Paper Class 12 2023-24 ISC PDF. To access the Class 12 Sample Paper, you can refer to the Selfstudys website as it is available in the portable document format. 

Exam Pattern in the ISC Class 12 Physical Education Sample Paper

The ISC Class 12 Sample Papers 2023-24 follows the specific pattern and rules. The same kind of rules are followed by the ISC Class 12 Physical Education Sample Paper. The Class 12 Physical Education Paper has two segments: theory Paper and practical Paper. The Class 12 Physical Education Specimen is just the replica of the actual exam which consists of various kinds of questions. 

Online or Offline

MCQ and Subjective Questions

2 to 3 hours

Nil

Hindi and English

Theory and Practical

Features of Physical Education Sample Paper Class 12 2023-24 ISC

You need to go through the Physical Education Sample Paper Class 12 2023-24 ISC so that you can get an idea about the past Paper questions and accordingly, you can prepare for the exam; those features are discussed below-

  • Divided into Different Sections: Questions in the ISC Class 12 Physical Education Sample Paper are divided into different sections so that you can give approximate time to solve each section. By completing Class 12 Physical Education questions within the given time, you can improve your time management skills. 
  • Answers are Given: After solving the questions, you can easily solve the doubts as it is already given in the ISC Class 12 Physical Education Sample Papers with answers. By solving Class 12 Physical Education doubts, you can enhance the two-way communication process. 
  • Set of Instructions are Given: In the Sample Paper Class 12 ISC Physical Education 2023-24, a set of instructions is given on the front so that you can follow them while completing of questions. Accordingly, you don’t miss out on any important information that can be relevant while solving Class 12 Physical Education questions. 
  • Marks are Distributed: For each question in the Class 12 ISC Physical Education Sample Paper, marks are distributed and accordingly, you need to present the answers. Through this, you can easily score well in the Class 12 Physical Education Specimen questions. 
  • Available in the Digital Format: The Class 12 Physical Education Specimen of ISC is available in the digital format so that you can access and solve questions from anywhere possible. This can help you to form a strong and firm foundation for Class 12 Physical Education. 
  • Time Frame to Complete: On the front page of the Class 12 Physical Education Specimen of ISC board, the time limit to complete the page is given; you need to complete the questions within the given time limit. 

How to Download the ISC Class 12 Physical Education Sample Paper?

To download the ISC Class 12 Physical Education Sample Paper, you can follow the given steps; those are-

  • Visit the Selfstudys website.

ISC Class 12 Physical Education Sample Paper 2023-24, ISC Class 12 Physical Education Sample Paper, ISC Class 12 Physical Education Sample Paper 2023-24 Solved, ISC Class 12 Physical Education Sample Paper PDF, Physical Education Sample Paper Class 12 2023-24 ISC PDF, Physical Education Sample Paper Class 12 2023-24 ISC, How to Download the ISC Class 12 Physical Education Sample Paper

  • Bring the arrow toward the State Books which can be seen in the navigation bar, and a drop-down menu will appear. 

ISC Class 12 Physical Education Sample Paper 2023-24, ISC Class 12 Physical Education Sample Paper, ISC Class 12 Physical Education Sample Paper 2023-24 Solved, ISC Class 12 Physical Education Sample Paper PDF, Physical Education Sample Paper Class 12 2023-24 ISC PDF, Physical Education Sample Paper Class 12 2023-24 ISC, How to Download the ISC Class 12 Physical Education Sample Paper

  • A drop-down menu will appear and select CISCE from the list. 

ISC Class 12 Physical Education Sample Paper 2023-24, ISC Class 12 Physical Education Sample Paper, ISC Class 12 Physical Education Sample Paper 2023-24 Solved, ISC Class 12 Physical Education Sample Paper PDF, Physical Education Sample Paper Class 12 2023-24 ISC PDF, Physical Education Sample Paper Class 12 2023-24 ISC, How to Download the ISC Class 12 Physical Education Sample Paper

  • Automatically a new page will appear, and select Sample Paper from the study material. 
  • Again a drop-down menu will appear, Select Physical Education from the list.
  • Once again a new page will appear, now you can go through the content of the Class 12 Specimen. 

Highlights of Physical Education Sample Paper Class 12 2023-24 ISC

There are a lot of details that are related to the Sample Paper Class 12 2023-24 ISC that need to be known. By knowing the Class 12 Specimen, you can easily cross-verify the content and move further in solving questions, Some of the details are discussed below-

  • CISCE authorities are the ones who upload the ISC Class 12 Physical Education Sample Paper on their official website. 
  • The Sample Paper Class 12 ISC Physical Education 2023-24 is available 24*7 on our website- Selfstudys which can be convenient and accessible. 
  • The Class 12 ISC Physical Education Sample Paper are arranged according to the exam pattern so that you can easily solve the actual question Paper. 
  • The Class 12 Physical Education board will be conducted during the month of March so before that you need to solve Specimen questions. 

CISCE

Class 12

Sample Question Papers of CISCE Class 12

National Level

India

CISCE

Council For The Indian School Certificate Examinations

Benefits of ISC Class 12 Physical Education Sample Paper

You should solve as many ISC Class 12 Physical Education Sample Paper as possible so that you can easily improve your score in the actual exam; some of the reasons are discussed below-

  • Provides an Examination Ambience: You can try completing the Physical Education Sample Paper Class 12 2023-24 ISC with time pressure and strict rules. Through the hypothetical situation, you are very much ready for the Class 12 Physical Education exam. 
  • Provides Practise: Through the Sample Paper Class 12 ISC Physical Education 2023-24, you can practice a vast amount of questions which can help you to form a solid understanding of the concepts and topics. 
  • Helps in Self-Evaluation: You can identify the areas that need improvement after solving the Class 12 ISC Physical Education Sample Paper. This can happen by cross-checking the answers of Class 12 Physical Education Specimen with the answer key or the concerned teacher. 
  • Helps to Learn the Exam Pattern: To get familiar with the format of the question Paper and types of questions, you can start solving the Class 12 Physical Education Specimen of the ISC board. By knowing the Class 12 Physical Education exam pattern, your preparation process can be more effective. 
  • Improves Time Management Skills: With time you can develop the habit of finishing the ISC Class 12 Physical Education Sample Papers with answers within the given time. If you are taking more time to answer the Class 12 Physical Education questions then you need to practise more.
  • Helps in Revision: Regular practice of Class 12 Physical Education questions from the ISC Specimen can help you to revise the chapters, topics, and concepts which is essential to scoring well. 

A Comprehensive Look to Solve the Physical Education Sample Paper Class 12 2023-24 ISC

You need to look towards the steps in a comprehensive way in order to solve the Physical Education Sample Paper Class 12 2023-24 ISC; some of the steps are discussed below-

  • Give First 15 Minutes to Understand: Give the first 15 minutes to read the instructions and questions given in the ISC Class 12 Physical Education Sample Paper. Through this, you can understand what is being asked in Class 12 Physical Education questions. 
  • Start With Easier or Known Ones: Try to start solving the Sample Paper Class 12 ISC Physical Education 2023-24 with easier or known ones so that you can build momentum and confidence with time. 
  • Don’t Spend Much Time: If you get stuck in any of the questions of the Class 12 ISC Physical Education Sample Paper, then it is advisable not to waste much time on one single question and move to the next. By solving other questions of Class 12 Physical Education, you can gain insight into challenging questions. 
  • Plan Your Answers: Before actually solving the Class 12 Physical Education Specimen of the ISC board, you can plan your answers by breaking them down into small and manageable steps. 
  • Present the Answers in a Logical Way: It is a must that you need to present the answers of Class 12 Physical Education in a logical way so that they can be presented neatly without much untidy. 
  • Review the Answers: You need to review after completing each and every question of ISC Class 12 Physical Education Sample Papers with answers so that there is a reduction in mistakes. 

How to Analyse the Mistakes After Solving ISC Class 12 Physical Education Sample Paper?

Analyzing your mistakes is the essential step after solving the ISC Class 12 Physical Education Sample Paper so that you can improve performance and understand the weaknesses; the steps to analyse are-

  • Review the Entire Physical Education Specimen: Go through the entire Physical Education Sample Paper Class 12 2023-24 ISC; note down the mistakes and make note of questions which you did incorrectly or have difficulty with. 
  • Identify the Mistakes: Categorise the mistakes of the Sample Paper Class 12 ISC Physical Education 2023-24 so that you can easily analyze it. Mistakes can be due to misunderstanding of Class 12 Physical Education concepts or misinterpretation of questions. 
  • Understand the Reasons: For each mistake in the Class 12 ISC Physical Education Sample Paper, try to understand the reason and cause. The reason or cause of the Class 12 Physical Education question can be due to lack of practice, insufficient understanding, or silly errors. 
  • Analyze the Approaches: Try to analyze the approaches of Class 12 Physical Education incorrect questions and understand the incorrect assumption or approach during the process. You can also try solving the same Class 12 Physical Education question again so that can understand the correct answer. 
  • Seek Guidance: If you are unable to rectify the mistakes of the Class 12 Physical Education Specimen of ISC then you can seek guidance from your concerned teachers or tutors; through this, you can easily analyze the mistakes. 
  • Practice Questions: To analyze the mistakes of ISC Class 12 Physical Education Sample Papers with answers, you need to practice different types of questions from textbooks or reference books. 

LPU University Admissions 2023 upto 100% Scholarship (All Pages)

  • NCERT Solutions for Class 12 Maths
  • NCERT Solutions for Class 10 Maths
  • CBSE Syllabus 2023-24
  • Social Media Channels
  • Login Customize Your Notification Preferences

isc physical education solved question paper

  • Second click on the toggle icon

isc physical education solved question paper

Provide prime members with unlimited access to all study materials in PDF format.

Allow prime members to attempt MCQ tests multiple times to enhance their learning and understanding.

Provide prime users with access to exclusive PDF study materials that are not available to regular users.

isc physical education solved question paper

Study Material

Entrance exams books

ISC Past Year Question Papers Class 12 | ISC Previous Year Solved Papers

Isc class 12 past year question papers.

ISC Class 12 Past Year Question Papers play a vital role in helping students in preparing for their Class 12 exams. We know their importance and hence, we have provided free and direct access to all the available ISC Class 12 Past Year Question Papers PDFs. Below, you can view subject-wise ISC Question Papers and their Marking Scheme Solutions (Analysis of Pupil Performance).

<red> → <red>ISC Class 12 English Language Previous Year Papers

<red> → <red>isc class 12 literature in english previous year papers, <red> → <red>isc class 12 hindi previous year papers, <red> → <red>isc class 12 mathematics previous year papers, <red> → <red>isc class 12 physics previous year papers, <red> → <red>isc class 12 chemistry previous year papers, <red> → <red>isc class 12 biology previous year papers, <red> → <red>isc class 12 accounts previous year papers, <red> → <red>isc class 12 business studies previous year papers, <red> → <red>isc class 12 commerce previous year papers, <red> → <red>isc class 12 economics previous year papers, <red> → <red>isc class 12 physical education previous year papers, <red> → <red>isc class 12 computer science previous year papers.

📈 Trending: 2024-25 ISC Class 12 Syllabus

📝 Recommended: ISC Class 12 Item Bank 2025

Past Year Question Papers are really important for students. They can:

  • Learn about the change in paper/ question pattern;
  • Practice all the important questions asked in the previous years; and
  • Understand how the marking scheme works for paper checkers so they can form answers to score maximum marks.

ISC's previous year's papers provide insights, practice, and a clear understanding of what to expect, ultimately aiding in effective exam preparation and performance. As the exam is nearing soon, try to make notes and practice as much as you can. We at Educart try to provide study materials like syllabus and marking scheme solutions so that a student can achieve their dream marks. 

1. How can I get the ISC Previous Year Paper Class 12 with Solutions for 2025 exams?

Students can directly download the ISC previous year papers for class 12 for the academic session 2024-25 from the CISCE’s official website- cisce.org and From ISC PYQs Class 12 . CISCE every year uploads the past year papers so that students can curate a plan accordingly to tackle all types of questions. 

2. When should we start solving ISC Previous Year Papers Class 12 for 2025 Exams?

Every class 12 student must practice the ISC previous year papers for the academic session 2024-25 at least 2-3 months prior to the board exams. Practicing past year papers will help them to jot down their weak and strong points, and also give them an idea about the actual board paper and the types of questions they could be asked.

Extra 10% Discount

isc physical education solved question paper

ISC Class 12 Previous Years Question Papers

Isc class 12 marking scheme, isc class 12 syllabus.

Buy Latest Books

Teacher's Corner

Main image

To Download PDF

Please verify your Whatsapp number first, so you can download this pdf immediately

Please type a valid 10 digit whatsapp number

isc physical education solved question paper

OTP sent, check your whatsapp

Your OTP is incorrect, Please enter valid OTP

isc physical education solved question paper

  • Mathematics (Standard)
  • Mathematics (Basic)
  • English L&L
  • English Communicative
  • Social Science
  • Information Technology
  • English Core
  • Mathematics
  • Accountancy
  • Business Studies
  • Political Science
  • Computer Application
  • Science (Hindi )
  • Maths (Hindi)
  • Social Science (Hindi )
  • Applied Maths
  • Physical Education
  • History & Civics
  • Literature in English
  • English Language
  • 10 Year Solved Papers
  • Class 10 Science
  • Class 10 Maths
  • Class 12 Physics
  • Class 12 Chemistry
  • Class 12 Maths
  • Class 12 Biology
  • Class 12 PCB Combo
  • Class 12 PCM Combo
  • Math Standard
  • Computer Applications
  • Class 10 English
  • Class 12 English

ISC Physical Education Specimen Paper 2024 (PDF) – CISCE Class 12 Physical Education Sample Paper

Isc board class 12 physical education sample paper 2024-25: this article will provide insights into the isc class 12 physical education sample paper and the answer key. keep reading the article to know..

Akshita Jolly

Instructions to Candidates

  • You are allowed an additional 15 minutes for only reading the question paper.
  • You must NOT start writing during reading time.
  • This question paper has nine printed pages and fifteen questions.
  • There are three sections in the paper: A, B and C. Internal choices have been provided in one question in Section B and one question in Section C.
  • Section A consists of one question having fifteen sub-parts of one mark
  • each. Attempt all questions.
  • Section B consists of five questions of five marks each. Attempt all questions.
  • Section C consists of nine questions of five marks each. Attempt all questions on any two games in this section.
  • The intended marks for questions are given in brackets [ ].

ISC Board Class 12 Physical Education Sample Paper 2024-25

   
   
   
   
(Recall)
   
   
   
   
   
   
   
   
   
   
   
   
   
   
   
   
   
   
   
   

This is the one section of the sample paper and if the students need the full sample paper, then they can download the free PDF from the link below.

CHECK: ISC Board Class 12 Physical Education Sample Paper 2024-25

Other Related Links

  • ISC Board Class 12 Environmental Science Sample Paper 2024-25: Download Sample Paper PDF For Free!
  • Madhya Pradesh MP Board Class 12 English Syllabus 2024-25: Download Syllabus PDF For Free!

Get here latest School , CBSE and Govt Jobs notification and articles in English and Hindi for Sarkari Naukari , Sarkari Result and Exam Preparation . Download the Jagran Josh Sarkari Naukri App .

  • India Post GDS Merit List 2024
  • India Post GDS Result 2024
  • UP Police Constable Admit Card 2024
  • UGC NET Exam Analysis 2024
  • UPSC Calendar 2025
  • India GDS Merit List 2024 PDF
  • UP Police Exam Analysis 2024 Live Updates
  • National Space Day Speech
  • National Space Day Essay
  • National Space Day Quiz

Latest Education News

A Cultural Odyssey: 30 Must-Visit Art Museums Across the USA

eShram Card: क्या है ई-श्रम कार्ड? लाभ, पात्रता और ऑनलाइन अप्लाई की सभी डिटेल्स यहां देखें, e-shram Card Download का तरीका

State-Wise Breakdown of Candidates Selected for India Post GDS Gramin Dak Sevak 2024 Post

Picture Puzzle IQ Test: Find the word “friends” in the birthday party scene in 5 seconds!

Brain Teaser: Which Glass Has More Water? Only 1% Pass This Test in 8 Seconds!

UP Police Exam Analysis 2024 Live Updates: Shift 1, 2 UPPRPB Constable Paper Review, Question Asked

NEET PG Result 2024 OUT: NBEMS Announces NEET Medical Entrance Exam Results at natboard.edu.in, Download Merit List PDF

Only 1% High IQ Can Spot 4 Differences in Popeye's Picture in 8 Seconds!

UP Police Constable Exam 2024 Live Updates: जानें 24 अगस्त को कैसा आया यूपी पुलिस कांस्टेबल का पेपर, क्या रहा कठिनाई स्तर, यहाँ देखें सभी अपडेट

UP Police Constable Question Paper 2024 {PDF} : यहाँ से डाउनलोड करें 24 अगस्त शिफ्ट 1, 2 के यूपी पुलिस कांस्टेबल प्रश्न पत्र PDF

MJPRU Result 2024 OUT at mjpruiums.in; Download UG and PG Marksheet PDF

Alagappa University DDE Result 2024 at alagappauniversity.ac.in; Direct Link to Download Marksheet PDF

IGNOU June TEE Result 2024 OUT at ignou.ac.in; Direct Link to Download Term End Exam UG and PG Grade Card

Osmania University Results 2024 OUT at osmania.ac.in; Direct Link to Download OU Marksheet PDF

Janmashtami 2024 Date: 25 or 26 August? Check Correct Date, Time, Fasting and More Details

Optical Illusion IQ Test: Can You Spot the Hidden Words in 8 Seconds?

Assam University Result 2024 OUT at ausexamination.ac.in; Direct Link to Download UG Provisional Marksheet

Personality Test: Your Finger Length Reveals Your Hidden Personality Traits

UP Police Constable Question Paper 2024: Download 24 August UPPRPB Papers Shift 1 and 2

UP Police Constable Answer Key 2024: Download UPPRPB Unofficial Papers for All Shifts

Robot

Download the Learning Outcomes App Today

Embibe Logo

Share this article

link

Table of Contents

Latest updates.

ISC Class 12 Compartment Exam 2022

ISC Class 12 Compartment Exam 2022

ISC Timetable for Class 12 2023

ISC Timetable for Class 12 2023

ISC Class 12 Syllabus: Download Chapter-wise PDFs

ISC Class 12 Syllabus: Download Chapter-wise PDFs

ISC Class 12 Books

ISC Class 12 Books

ISC Class 12 Question Papers: Check Details

ISC Class 12 Question Papers: Check Details

ISC Class 12 Model Question Papers 2023

ISC Class 12 Model Question Papers 2023

ICSE Class 10 Result 2023: Steps to Check Results

ICSE Class 10 Result 2023: Steps to Check Results

ISC Board Class 12 Study Material 2023

ISC Board Class 12 Study Material 2023

ISC Board Class 12 Solutions: Access ICSE Class 12 Papers With Solutions

ISC Board Class 12 Solutions: Access ICSE Class 12 Papers With Solutions

ISC Board Class 12 Mock Test 2024: Practice Online Test Series

ISC Board Class 12 Mock Test 2024: Practice Online Test Series

Tag cloud :.

  • entrance exams
  • engineering
  • ssc cgl 2024
  • Written By Shobha_M
  • Last Modified 28-03-2024

ISC Class 12 Previous Year Question Papers: Download PDFs

ISC Class 12 Previous Year Question Papers: Council for the Indian School Certificate Examinations (CISCE) conducts the Indian School Certificate Examinations (ISC) for Class 12 students. The exams take place in two semesters – Semester 1 and Semester 2. ISC Class 12 exams are an important stepping stone in students’ career and future job prospects.

Practice ISC Class 12 Exercise Questions with Hints & Solutions Here! Pro Tip:  At Embibe, we believe that all students have the right to personalised guidance. Embibe offers personalised attention to all the ISC Class 12 students to help them prepare for their exams and score good marks.
Mathematics
Physics
Biology
Chemistry

Students preparing for the 12th Class exams should thoroughly know the exam pattern, syllabus, and marking scheme. Once students have finished the ISC Class 12 Syllabus, they should diligently solve the previous year’s question papers for ISC Class 12. Solving question papers are a great way to boost exam confidence and equip students with crucial exam-taking skills.

ISC Class 12 Exams: Overview

The ISC Class 12 Semester 2 exams shall commence on April 25 2022, until June 6, 2022. Students must know the detailed syllabus, exam pattern and generic exam details before taking the semester-end exams. Check the below table to understand all the vital information about the ISC Class 12 exams:

Indian School Certificate (ISC)
Council for the Indian School Certificate Examinations (CISCE)
1958
English
Percentage/Marks
Council for the Indian School Certificate Examinations (CISCE)P 35-36, Sector VIPushp Vihar, Saket, New Delhi – 110017
(011) 29564831/33/37
(011) 29564735
National level
Offline via the school for regular students and online for private students
Offline
English, Hindi, Telugu, Kannada, Sanskrit, Tamil, Malayalam, Urdu, Bengali, Marathi, Gujarati, Punjabi etc.
Mathematics, Biology, Physics, Chemistry, Computer Studies, Social Studies, Economics, Accountancy, Political science etc.
Twice in a year
Test your ISC Class 12 Concepts with Unlimited Practice Questions ! Pro Tip:  At Embibe, we believe that all students have the right to personalised guidance. Embibe offers personalised attention to all the ISC Class 12 students to help them prepare for their exams and score good marks.
Mathematics
Physics
Chemistry
Biology

ISC Class 12 Question Papers 2022: Download PDFs

Solving previous years’ question papers for ISC Class 12th has numerous benefits. Students get an idea of the actual 12th Class board exams. When they attempt these question papers at a stretch of 3 hours, they understand their learning progress and take effective measures accordingly. Keep reading this article to collect all the previous year’s question papers for ISC Class 12 starting from the academic session 2021-2022. 

ISC Class 12 Previous Year Question Papers 2020

Click on the below links to download subject-wise previous year question papers for ISC Class 12 2020:

ISC Class 12 English Paper 1 Question Paper
ISC Class 12 English Paper 2 Question Paper
ISC Class 12 Hindi Question Paper
ISC Class 12 Elective English Question Paper
ISC Class 12 History Question Paper
ISC Class 12 Political Science Question Paper
ISC Class 12 Geography Question Paper
ISC Class 12 Sociology Question Paper
ISC Class 12 Psychology Question Paper
ISC Class 12 Economics Question Paper
ISC Class 12 Commerce Question Paper
ISC Class 12 Accounts Question Paper
ISC Class 12 Business Studies Question Paper
ISC Class 12 Mathematics Question Paper
ISC Class 12 Physics Paper 1 Question Paper
ISC Class 12 Physics Paper 2 Question Paper
ISC Class 12 Chemistry Paper 1 Question Paper
ISC Class 12 Chemistry Paper 2 Question Paper
ISC Class 12 Biology Paper 1 Question Paper
ISC Class 12 Biology Paper 2 Question Paper
ISC Class 12 Home Science Paper 1 Question Paper 1
ISC Class 12 Home Science Paper 2 Question Paper
ISC Class 12 Fashion Designing Question Paper 1
ISC Class 12 Fashion Designing Question Paper 2
ISC Class 12 Electricity and Electronics Question Paper
ISC Class 12 Computer Science Question Paper 1
ISC Class 12 Computer Science Question Paper 2
ISC Class 12 Geometrical and Mechanical Drawing Question Paper
ISC Class 12 Geometrical and Building Drawing Question Paper
ISC Class 12 ART Question Paper 1
ISC Class 12 ART Question Paper 2
ISC Class 12 ART Question Paper 3
ISC Class 12 ART Question Paper 4
ISC Class 12 ART Question Paper 5
ISC Class 12 IMH Question Paper 1
ISC Class 12 IMH Question Paper 2
ISC Class 12 Indian Music Carnatic Question Paper 1
ISC Class 12 Indian Music Carnatic Question Paper 2
ISC Western Music Theory Question Paper 1
ISC Western Music Practical Question Paper 2
ISC Class 12 Physical Education Question Paper
ISC Class 12 Environmental Science Question Paper
ISC Class 12 Biotechnology Paper 1
ISC Class 12 Biotechnology Paper 2
ISC Class 12 Mass Media & Communication Question Paper

ISC Class 12 Previous Year Question Papers 2019

Click on the below links to download subject-wise previous year question papers for ISC Class 12 2019:

ISC Class 12 English Paper 1 Question Paper
ISC Class 12 English Paper 2 Question Paper
ISC Class 12 Hindi Question Paper
ISC Class 12 Elective English Question Paper
ISC Class 12 History Question Paper
ISC Class 12 Political Science Question Paper
ISC Class 12 Geography Question Paper
ISC Class 12 Sociology Question Paper
ISC Class 12 Psychology Question Paper
ISC Class 12 Economics Question Paper
ISC Class 12 Commerce Question Paper
ISC Class 12 Accounts Question Paper
ISC Class 12 Business Studies Question Paper
ISC Class 12 Mathematics Question Paper
ISC Class 12 Physics Paper 1 Question Paper
ISC Class 12 Physics Paper 2 Question Paper
ISC Class 12 Chemistry Paper 1 Question Paper
ISC Class 12 Chemistry Paper 2 Question Paper
ISC Class 12 Biology Paper 1 Question Paper
ISC Class 12 Biology Paper 2 Question Paper
ISC Class 12 Home Science Paper 1 Question Paper 1
ISC Class 12 Home Science Paper 2 Question Paper
ISC Class 12 Fashion Designing Question Paper 1
ISC Class 12 Fashion Designing Question Paper 2
ISC ELE and Electronics Question Paper
ISC Computer Science Question Paper 1
ISC Class 12 Computer Science Question Paper 2
ISC Class 12 Geometrical and Mechanical Question Paper 1
ISC Class 12 ART Question Paper 1
ISC Class 12 ART Question Paper 2
ISC Class 12 ART Question Paper 3
ISC Class 12 ART Question Paper 4
ISC Class 12 ART Question Paper 5
ISC Class 12 IMH Question Paper 1
ISC Class 12 IMH Question Paper 2
ISC Class 12 Indian Music Carnatic Question Paper 1
ISC Class 12 Indian Music Carnatic Question Paper 2
ISC Class 12 Western Music Theory Question Paper
ISC Class 12 Western Music Practical Question Paper
ISC Class 12 Physical Education Question Paper
ISC Class 12 Environment Question Paper
ISC Class 12 Biotechnology Paper 1
ISC Class 12 Biotechnology Paper 2

ISC Class 12 Previous Year Question Papers 2018

Click on the below links to download subject-wise previous year question papers for ISC Class 12 2018:

ISC Class 12 English Paper 1 Question Paper
ISC Class 12 English Paper 2 Question Paper
ISC Class 12 Hindi Question Paper
ISC Class 12 Elective English Question Paper
ISC Class 12 History Question Paper
ISC Class 12 Political Science Question Paper
ISC Class 12 Geography Question Paper
ISC Class 12 Sociology Question Paper
ISC Class 12 Psychology Question Paper
ISC Class 12 Economics Question Paper
ISC Class 12 Commerce Question Paper
ISC Class 12 Accounts Question Paper
ISC Class 12 Business Studies Question Paper
ISC Class 12 Mathematics Question Paper
ISC Class 12 Physics Paper 1 Question Paper
ISC Class 12 Physics Paper 2 Question Paper
ISC Class 12 Chemistry Paper 1 Question Paper
ISC Class 12 Chemistry Paper 2 Question Paper
ISC Class 12 Biology Paper 1 Question Paper
ISC Class 12 Biology Paper 2 Question Paper
ISC Class 12 Home Science Paper 1 Question Paper 1
ISC Class 12 Home Science Paper 2 Question Paper
ISC Class 12 Fashion Designing Question Paper 1
ISC Class 12 Fashion Designing Question Paper 2
ISC ELE and Electronics Question Paper
ISC Computer Science Question Paper 1
ISC Class 12 Computer Science Question Paper 2
ISC Class 12 Geometrical and Mechanical Question Paper 1
ISC Class 12 ART Question Paper 1
ISC Class 12 ART Question Paper 2
ISC Class 12 ART Question Paper 3
ISC Class 12 ART Question Paper 4
ISC Class 12 ART Question Paper 5
ISC Class 12 IMH Question Paper 1
ISC Class 12 IMH Question Paper 2
ISC Class 12 Indian Music Carnatic Question Paper 1
ISC Class 12 Indian Music Carnatic Question Paper 2
ISC Western Music Theory Question Paper
ISC Western Music Practical Question Paper
ISC Class 12 Physical Education Question Paper
ISC Class 12 Environment Question Paper
ISC Class 12 Biotechnology Paper 1
ISC Class 12 Biotechnology Paper 2

ISC Class 12 Previous Year Question Papers 2017

Click on the below links to download subject-wise previous year question papers for ISC Class 12 2017:

ISC Class 12 English Paper 1 Question Paper
ISC Class 12 English Paper 2 Question Paper
ISC Class 12 Hindi Question Paper
ISC Class 12 Elective English Question Paper
ISC Class 12 History Question Paper
ISC Class 12 Political Science Question Paper
ISC Class 12 Geography Question Paper
ISC Class 12 Sociology Question Paper
ISC Class 12 Psychology Question Paper
ISC Class 12 Economics Question Paper
ISC Class 12 Commerce Question Paper
ISC Class 12 Accounts Question Paper
ISC Class 12 Business Studies Question Paper
ISC Class 12 Mathematics Question Paper
ISC Class 12 Physics Paper 1 Question Paper
ISC Class 12 Physics Paper 2 Question Paper
ISC Class 12 Chemistry Paper 1 Question Paper
ISC Class 12 Chemistry Paper 2 Question Paper
ISC Class 12 Biology Paper 1 Question Paper
ISC Class 12 Biology Paper 2 Question Paper
ISC Class 12 Home Science Paper 1 Question Paper
ISC Class 12 Home Science Paper 2 Question Paper
ISC Class 12 Fashion Designing Question Paper 1
ISC Class 12 Fashion Designing Question Paper 2
ISC ELE and Electronics Question Paper
ISC Computer Science Question Paper 1
ISC Class 12 Computer Science Question Paper 2
ISC Class 12 Geometrical and Mechanical Question Paper 1
ISC Class 12 ART Question Paper 1
ISC Class 12 ART Question Paper 2
ISC Class 12 ART Question Paper 3
ISC Class 12 ART Question Paper 4
ISC Class 12 ART Question Paper 5
ISC Class 12 IMH Question Paper 1
ISC Class 12 IMH Question Paper 2
ISC Class 12 Indian Music Carnatic Question Paper 1
ISC Class 12 Indian Music Carnatic Question Paper 2
ISC Western Music Theory Question Paper
ISC Western Music Practical Question Paper
ISC Class 12 Physical Education Question Paper
ISC Class 12 Environment Question Paper
ISC Class 12 Biotechnology Paper 1
ISC Class 12 Biotechnology Paper 2

How to Download ISC 12 Previous Year Papers?

Students can download ISC Class 12 Questions Papers starting from the year 2017-2020 from this article. Alternatively, students can download the previous year’s question papers from the official website. Follow the below instructions:

  • 1st Step: Log on to the official website cisce.org. 
  • 2nd Step: Click on the ‘Downloads’ option from the tab.
  • 3rd Step: Students will get various options of ‘ISC Question Papers’ year-wise. 
  • 4th Step: Click on the desired year option from 2017 to 2020.
  • 5th Step: A zip file for the same year will be downloaded into your laptops/PC. 
  • 6th Step: Extract the files to access subject-wise ISC Class 12 question papers. 

Advantages of Solving ISC Question Papers for Class 12

Solving previous years’ question papers has numerous advantages. Let us take a look at some of the major benefits:

  • Solving ISC question papers give the student an idea about which kind of chapter questions are repetitive. They can accordingly focus on those important chapters. 
  • Students identify their weak and strong portions of the ISC 12th Class syllabus. 
  • When students solve the ISC question papers in one sitting of 3 hours, they learn how to manage time efficiently for the real-time exams. 
  • Students also polish their writing and presentation skills. Students learn how to answer neatly and practice important diagrams.
  • Lastly, solving previous year’s question papers is the best way to revise for 12th Class Semester 2 exams and boos one’s exam confidence. 

Now that students have access to all the ISC Class 12 Question Papers, know the importance of solving them and how to download them, they should go through the exam pattern and syllabus. Knowing the syllabus and exam pattern is a crucial step for exam preparation. Keep reading to know all the important exam details and the 12th Grade syllabus. 

ISC Class 12 Exam Pattern 2022

The ISC Class 12 Semester 2 exams 2022 shall occur for theory and practical. Students can generally expect short answers, long answers, and value-based questions. Knowing the exam pattern, marking scheme, and syllabus can help students prepare for the board exams 2022 strategically. Check the below table to get an idea about the subject-wise exam pattern and marking scheme for ISC 12th Class exams:

Composition, directed writing, short answer questions based on grammar and comprehension100
One textual question (compulsory) on the Shakespeare play/alternative prescribed play together with four other questions on at least three texts, which may include the Shakespeare play/alternative play.100
Composition, comprehension, grammar and questions from prescribed textbooks100
Short composition, passage, unseen passage and questions and prescribed textbooks100
Questions on grammar, unseen translation and translation from the prescribed textbooks100
Questions are based on prose, drama and poetry100
Questions are asked from Algebra, Calculus, Vectors, probability, and Linear Programming among others100
Questions are based on definitions of fashion, design details, wardrobe planning, and designers70
Compulsory short answer questions and long questions. The topics are based on the distribution of electric power, D.C. generator and motor, A.C. motor, wires, cables, and electrical wires.100 
Short answer questions and long questions are based on friction, limiting friction, machines, power, and momentum.100 
Compulsory short answer questions and there are more sections, where questions are asked from Boolean Algebra, Computer Hardware, Implementation of algorithms to solve problems, etc.70 
Candidates are required to answer all questions. The preparation of working drawings and assemblies from dimensioned sketches based on the following- fastening (nuts, bolts, studs, keys, cotters, pins, locking devices), rigid and flexible joints, screw threads wheels and transmission of motion and power100 
The questions are based on building drawing should be based on the form and construction of simple buildings and parts of buildings; small dwelling houses (single and two stories), garages, sheds and greenhouses.100 
Drawing or painting from still life, drawing and painting from nature, drawing and painting of a living person100
The syllabus is divided into three parts Vocal, Instrumental and Tabla. Candidates need to choose one of them70 
The theory paper is divided into two sections A and B. Candidates are required to answer five questions out of seven from Section A, each carrying 8 marks. Section B is based on questions on major games in the syllabus.70 
Compulsory short questions answer and other questions are based on human beings and nature, Population and Conservation Ecology and monitoring population70 

ISC Class 12 Exams 2022: Check Subject-wise Syllabus

Before making a detailed study plan for ISC Class 12 Semester 2 exams, students should know the syllabus and chapter-wise weightage in detail. It will help them understand the subjects and important topics covered for the academic session. Students can decide how much time to devote to each subject and which portions of the syllabus they need to study first. 

Click on the below table to get access to the ISC Class 12 syllabus for all subjects:

FAQs on ISC Class 12 Question Papers

Q1. Is ISC board tough? Ans. No, the ISC board is not that difficult to score well. Even though students feel that the syllabus is vast with many topics to cover, you can score good marks with diligent exam preparation.

Q2. Is the syllabus for ISC Class 12 2022 reduced? Ans. Yes, CISCE has reduced the ISC Class 12 syllabus 2021-2022.

Q3. How many hours should I study for ISC 12th Class? Ans : Students should devote a minimum of 4-5 hours daily for ISC Class 12 exam preparation.

Q4. What is the importance of solving ISC Class 12 previous year question papers? Ans. By solving ISC Class 12 previous year question papers, students get an idea of the actual 12th Class board exams. When they attempt these question papers at a stretch of 3 hours, they understand their learning progress and take effective measures accordingly. 

Q5. What are the exam tips to keep in mind while attempting ISC Class 12 exam 2022? Ans . Students should bear in mind the following tips: 

  • Do not leave any questions unattempted.
  • Attempt the ones that you are sure of first.
  • Try to support your long answers with diagrams and charts.
  • Always leave spaces between answers.
  • Read the question paper carefully before attempting the answers. 

Now that students have the ISC previous year question papers for Class 12, they can begin their exam preparation. Try to solve as many question papers and sample papers for semester 2 2022 exams. Students can also check the crucial videos for ISC Class 12 . Subject matter experts have designed these videos to provide vital conceptual clarity and provide extra knowledge about various Class 12 topics. 

Related Articles

ISC Class 12 Compartment Exam: The Indian School Certificate (ISC) Class 12 compartment exam is scheduled to be conducted in August 2022 (tentatively). The Council...

ISC Timetable for Class 12: The Council For The Indian School Certificate Examinations (CISCE) has released the 12th timetable on December 1, 2022. According to...

ISC Class 12 Syllabus: The syllabus forms the basic step for preparing for the board exam. The topics studied at this level are crucial for...

ISC Class 12 Books: The Council for Indian School Certificate Examinations (CISCE) designs the academic curriculum for ISC Class 12. The syllabus is prepared to introduce...

ISC Class 12 Question Papers: Getting a clear understanding of the exam syllabus is one of the best ways to score higher marks in the...

ISC Class 12 Model Question Papers: The Council for Indian School Certificate Examinations (CISCE) conducts the Indian School Certificate or ISC exams for Class 12...

ICSE Class 10 Result 2023: The Council for the Indian School Certificate Examinations (CISCE) declares the results for ICSE Class 10 usually in May. Students...

Are you scrolling through your phone to find the proper online books and previous year's question papers for ISC Class 12 exam preparation? Well, we...

We understand the constraints and the competition you face while preparing for your ISC Class 12 examination. This is why we at Embibe have made...

If you are studying in ISC Board Class 12, then you will know that your success depends largely on the preparation you have put in....

ISC Board Class 12 Chapters 2023

Are you a student of Class 12 and worried about acing your board exams? With the board exams approaching, every student is giving their heart...

12th ICSE Topics

Remember the time when kindergarten was a time to play around? Now, students look back to that time and wonder how to make learning fun...

ISC Syllabus for Class 11: Download Latest Syllabus PDF

ISC Syllabus for Class 11: CISCE decides the syllabus for ISC Class 11. It is significantly necessary for the students to go through the syllabus...

ICSE Class 9 Sample Papers 2023

"Success is not the key to happiness. Happiness is the key to success." - Albert Schweitzer. As students of Class 9 prepare for their ICSE...

ISC Books for Class 11: Important Books for Exams

ISC Books for Class 11: Students appearing for ISC exams need to ensure that they closely follow the ISC books for Class 11 for their...

ISC Class 11 Exam Chapters

Believe you can, and you’re halfway there! If you are a passionate student aiming to achieve the best results in ISC Class 11 final exams,...

ISC Class 11 Exam Study Material 2023

Do you wish to be different and unique in every way, from styling your clothes to your personality and how you conduct yourself? I guess...

ISC Class 11 Model Question Paper: Download PDF

ISC Class 11 Model Question Paper: Class 11 is considered as a foundation step in the career-building journey as students opt for Science, Arts, Commerce...

ISC Class 12 Admit Card 2023

ISC Class 12 Admit Card: The Council for the Indian School Certificate Examinations (CISCE) will tentatively release the ISC Class 12 admit card soon. Students...

ISC Board Class 12 Preparation Tips 2023

12th ISC Board Preparation Tips: The Council for the Indian School Certificate Examinations conducts the 12th ISC board examination every year. Class 12 is an...

ICSE Syllabus for Class 8 Chemistry 2023

ICSE Syllabus for Class 8 Chemistry: The ICSE Class 8 Syllabus is created by ICSE board officials. Chemistry is an important branch of science because...

ICSE Books for Class 5: Check Subject-wise Books

ICSE Books for Class 5: Students can learn a lot from the ICSE Class 5 syllabus. The curriculum for all disciplines includes themes from English,...

ICSE Schools in Chennai: Check Details

ICSE Schools in Chennai: In this article, students can find a list of the top 15 ICSE Schools in Chennai. There are several top schools...

List of Top ICSE Schools in Kolkata 2022-23

Since the 1960s, Kolkata has gained a reputation as the country's unofficial literary capital as it is home to some of the most celebrated Indian...

ISC Board Class 12 Exam Pattern

ISC Board Class 12 Exam Pattern: CISCE The Council for Indian School Certificate Examination regulates the Indian School Certificate (ISC) exams for Class 12. The...

12th ISC Books

Every year, around 1 lakh students appear for the Indian School Certificate (ISC) Board Class 12 exams, and only 35-40% of the students score more...

ISC Class 12: Check Reduced Syllabus, Exam Date and Exam Pattern

ISC Class 12 Exam: The Indian School Certificate or ISC exam is the annual examination conducted for Class 12 students enrolled in CISCE-affiliated schools. The...

isc physical education solved question paper

39 Insightful Publications

World Economic Forum

Embibe Is A Global Innovator

accenture

Innovator Of The Year Education Forever

Interpretable And Explainable AI

Interpretable And Explainable AI

Tedx

Revolutionizing Education Forever

Amazon AI Conclave

Best AI Platform For Education

Forbes India

Enabling Teachers Everywhere

ACM

Decoding Performance

World Education Summit

Leading AI Powered Learning Solution Provider

Journal of Educational Data Mining

Auto Generation Of Tests

BW Disrupt

Disrupting Education In India

Springer

Problem Sequencing Using DKT

Fortune India Forty Under Fourty

Help Students Ace India's Toughest Exams

Edtech Digest

Best Education AI Platform

Nasscom Product Connect

Unlocking AI Through Saas

Tech In Asia

Fixing Student’s Behaviour With Data Analytics

Your Story

Leveraging Intelligence To Deliver Results

City AI

Brave New World Of Applied AI

vccircle

You Can Score Higher

INK Talks

Harnessing AI In Education

kstart

Personalized Ed-tech With AI

StartUpGrind

Exciting AI Platform, Personalizing Education

Digital Women Award

Disruptor Award For Maximum Business Impact

The Mumbai Summit 2020 AI

Top 20 AI Influencers In India

USPTO

Proud Owner Of 9 Patents

StartUpGrind

Innovation in AR/VR/MR

StartUpGrind

Best Animated Frames Award 2024

Close

Trending Searches

Previous year question papers, sample papers.

Unleash Your True Potential With Personalised Learning on EMBIBE

Pattern

Ace Your Exam With Personalised Learning on EMBIBE

Enter mobile number.

By signing up, you agree to our Privacy Policy and Terms & Conditions

ISC Physical Education Semester-1 Specimen Paper Solved Class -12

ISC Physical Education Semester-1 Specimen Paper Solved Class -12 for practice.  Step by step solutions of ISC Class-12 specimen model sample paper .  During solutions of semester-1  Physical Education specimen paper we explain with figure , graph, table whenever necessary so that student can achieve their goal in next upcoming exam of council. Visit official website CISCE  for detail information about ISC Board Class-12.

Board ISC
Class  12th (XII)
Subject Physical Education
Topic Semester-1 ISC Specimen Paper Solved
Syllabus  on bifurcated syllabus (after reduction)
session 2021-22
Question Type  MCQ/ Objective (as prescribe by council)
Total
question
Total 70
Max
mark
70

ISC Physical Education Semester-1 Specimen Paper Solved  2022 Class-12

Physical development for the Romans was:

(a) Military motive (b) All round development (c) Recreational motive (d) None of the above Answer-

(a) Military motive

In which country developed the concept of all round development?

(a) Rome (b) Germany (c) Denmark (d) Greece Answer-

(c) Denmark

Per Henrick Ling had established The Royal Institute of Gymnastics, where students were trained in three different phases. Which of following is not one of the phases?

(a) Medical Gymnastics (b) Educational Gymnastics (c) Military Gymnastics (d) Professional Gymnastics Answer-

(d) Professional Gymnastics

Which God was honoured by the Greeks in their famous Olympian festival?

(a) Poseidon (b) Zeus (c) Aphrodite (d) Apollo Answer-

In which year did India host the biggest games festival : The Asian Games?

(a) 1948 (b) 1950 (c) 1951 (d) 1954 Answer-

During which period were Arrow shooting, Chariot racing and Hunting prevalent in India?

(a) Vedic Period (b) Medieval Period (c) Ancient Period (d) Indus Valley Civilisation Answer-

(a) Vedic Period

Question 7  ISC Physical Education Semester-1 Specimen

Physical education develops qualities like- Patience, ____________, Sympathy and Tolerance.

(a) Rebellious (b) Co-operation (c) Jealousy (d) Ruthlessness Answer-

(b) Co-operation

Games and sports are a means of:

(a) National and International integration. (b) Increasing the divide. (c) Starting street fights. (d) Initiating political protests. Answer-

(a) National and International integration.

Which of the following traits reflects personality development?

(a) Helpfulness (b) Team spirit (c) Both (a) and (b) (d) None of them Answer-

(c) Both (a) and (b)

Question 10

Sports _______ people of all castes, creed and religion.

(a) Unite (b) Differentiates (c) Divide (d) Distance Answer-

Question 11

Games and Sports develop individuals by:

(a) Increasing Physical activity levels (b) Promoting healthy attitude and behaviour (c) Both (a) and (b) (d) None of them Answer-

Question 12

Sports in the present time aim at:

(a) Living a fuller life. (b) Earning a livelihood. (c) All round development of human beings. (d) Just as a pass time activity. Answer-

(c) All round development of human beings.

Question 13

Which method is used to draw the fixture of a single league tournament?

(a) Cyclic method (b) Tabular method (c) Staircase method (d) All of the above Answer-

(d) All of the above

Question 14

Who developed Interval training Method?

(a) Woldemar Gerschler and Dr. Hans Reindell (b) Dr. Harre and Dr. Martin (c) Gosta Holmer (d) Dr. V. Aaken Answer-

(c) Gosta Holmer

Question 15

When and where was Netaji Subhash National Institute of Sports founded?

(a) Banglore 1962 (b) Kolkata 1961 (c) Thiruvananthapuram 1962 (d) Patiala 1961 Answer-

(d) Patiala 1961

Question 16

____________ tournament is set up to provide an opportunity for a second chance in a tournament for those participants who lose their first match.

(a) Single league (b) Double league (c) Double counter league (d) Single counter league. Answer-

(b) Double league

Question 17

Who founded the Indian Olympic Association (IOA) in 1927?

(a) Eric Brandon (b) Sir B P Nair (c) Sir Dorabji Tata (d) John Brown Answer-

(c) Sir Dorabji Tata

Question 18

Isotonic contraction, which means equal tension, is known as:

(a) Eccentric contraction. (b) Static contraction. (c) Concentric contraction. (d) Dynamic contraction. Answer-

(d) Dynamic contraction.

Question 19

When an isotonic exercise is performed against resistance the load remains

(a) Constant. (b) Variable. (c) Static. (d) Oscillating. Answer-

(c) Static.

Question 20

Which of the following is not an objective of the sports training?

(a) Technique development (b) Aerobic endurance training (c) Physical fitness (d) Tactical development Answer-

(b) Aerobic endurance training

Question 21

What is the rate of speed, at which physical activity is performed, known as?

(a) Work-load. (b) Exercise volume. (c) Load stimulus. (d) Intensity. Answer-

(d) Intensity.

Question 22

Which of the following techniques stresses on “programmed phase of work and recovery”?

(a) Interval training. (b) Continuous method. (c) Fartlek training. (d) Acceleration runs. Answer-

(a) Interval training.

Question 23

What is the central core of circuit training?

(a) Exercise intensity. (b) Exercise density. (c) Exercise volume. (d) Exercise continuity. Answer-

(d) Exercise continuity.

Question 24  ISC Physical Education Semester-1 Specimen

Players particularly benefit from warming exercise because:

(a) The cardio-respiratory system gets ready for the ensuing action. (b) They gain a lot with little effort. (c) Running incorporates only a limited range of movements. (d) This accelerates their peripheral blood supply. Answer-

(a) The cardio-respiratory system gets ready for the ensuing action.

Question 25

Circuit training method was developed by:

(a) G.D. Sondhi and Mr. Dorabji Tata. (b) Dr. D.G. Noehrem. (c) Mr. Dorabji Tata. (d) R.E. Morgan and G.T. Adamson. Answer-

(d) R.E. Morgan and G.T. Adamson.

Question 26

Which of the following is not a type of strength training?

(a) Maximum strength. (b) Knock-out strength (c) Explosive strength. (d) Strength endurance.

(b) Knock-out strength

Question 27

Which of the following rules is not related to weight training?

(a) Concentrating on the muscles during exercising. (b) Maintaining a good rhythm during exercise. (c) Resting between the sets for 30 to 60 seconds. (d) Not warming up and stretching before the workouts. Answer-

(d) Not warming up and stretching before the workouts.

Question 28

What is the advantage of Cooling Down?

(a) Abnormal blood circulation. (b) Efficient work of the bodily system to work efficiently. (c) Increases in level of Adrenaline in the blood. (d) Proper supply of blood and oxygen to muscles. Answer-

(d) Proper supply of blood and oxygen to muscles.

Question 29

Which of the following is not a component of Physical fitness?

(a) Agility (b) Anaerobic capacity (c) Lexibility (d) Muscle composition Answer-

(c) Lexibility

Question 30

Which of the following is not a Weight Training exercise? (a) Shoulder press (b) Bench press (c) Running (d) Triceps press Answer-

(c) Running

Question 31

Fartlek, which means “speed play” is a variation of

(a) Fast continuous method. (b) Interval method. (c) Acceleration runs. (d) Circuit training. Answer-

(a) Fast continuous method.

Question 32

Which of the following in an incorrect method of training?

(a) Interval training method. (b) Continuous method. (c) Explosive strength method. (d) Fartlek method Answer-

(c) Explosive strength method.

Question 33

What does full form of LNIPE stand for:

(a) Lakshmibai National Institute of Physical Education. (b) Lakshmibai National University of Physical Education. (c) Lakshmi National Institute of Physical Education. (d) Lakshmibai National Collage of Physical Education. Answer-

(a) Lakshmibai National Institute of Physical Education.

Question 34

In which year was the LNIPE founded?

(a) 1957 (b) 1958 (c) 1857 (d) 1956 Answer-

Question 35

Give the full form of YMCA:

(a) Young Men’s Christian Authority. (b) Young Men’s Christian Association. (c) Youth Men’s Christian Association. (d) Youth Men’s Challenge Association. Answer-

(b) Young Men’s Christian Association.

Question 36

Who founded the YMCA college of Physical Education?

(a) Mr. Harry Crowe Buck (b) Ministry of Youth Affairs and Sports (c) Rev. J.H. Messmore (d) SAI Answer-

(a) Mr. Harry Crowe Buck

Question 37

IOA stands for:

(a) International Olympic Authority (b) Indian Olympic Authority (c) Indian Olympic Association (d) International Olympic Association Answer-

(c) Indian Olympic Association

Question 38  ISC Physical Education Semester-1 Specimen

Which one of the following option is not a purpose of the IOC?

(a) To ensure the regular celebration of the Olympic games. (b) To lead the fight against doping in sports. (c) Development of sports for all. (d) To not take action to strengthen the unity of the Olympic movement. Answer-

(d) To not take action to strengthen the unity of the Olympic movement.

Question 39

When was International Olympic Committee formed?

(a) 26 June 1896 (b) 27 June 1896 (c) 26 June 1898 (d) 25 June 1894 Answer-

(d) 25 June 1894

Question 40

What does NSNIS stand for?

(a) Netaji Sports National Institute of Survey. (b) Netaji Subhas National Institute of Sports. (c) New Sports National Institute of Sports. (d) Netaji Subhas National Indian Sports. Answer-

(b) Netaji Subhas National Institute of Sports.

Question 41

Where is the permanent headquarter of IOC located?

(a) In Atlanta (U.S.A) (b) In Lausanne (Switzerland) (c) In Beijing (China) (d) In Stockholm (Sweden) Answer-

(b) In Lausanne (Switzerland)

Question 42

The Olympic motto consists of Latin words:

(a) Situs, Altius, and Forties. (b) Citeus, Altius and Forties. (c) Citius, Altius, and Fortius. (d) None of above. Answer-

(c) Citius, Altius, and Fortius.

Question 43

What does the Olympic flag consist of?

(a) A Green background with no border and Olympic symbol in the centre. (b) A Red background with no Olympic symbol. (c) A White background with no border and Olympic symbol in the centre. (d) None of the above. Answer-

(c) A White background with no border and Olympic symbol in the centre.

Question 44

When was Sports Authority of India formed?

(a) 1982 (b) 1984 (c) 1983 (d) 1988 Answer-

Question 45

Who is the first president of IOA?

(a) Sir Dorabji Tata (b) G.D Sodhi (c) Raja Bhalender Singh (d) Suresh Kalmadi Answer-

(a) Sir Dorabji Tata

Question 46

Which of the following pairs is not correct?

(a) NSNIS -1961 (b) YMCA -1920 (c) SAI -1984 (d) IOA -1999 Answer-

(d) IOA -1999

Question 47

Who is the Father of Modern Olympics games?

(a) Demetrius Vikelas (b) Le Marques Samaranch (c) Baron Pierre de Cubertin (d) Ferenc Kemeny Answer-

(c) Baron Pierre de Cubertin

Question 48

The Five rings of the Olympic flag are in five different colours. They are red, green, yellow, blue and ________.

(a) Orange (b) Indigo (c) Black (d) Violet Answer-

Question 49

In which year was the first summer Olympics held?

(a) 1896 (b) 1897 (c) 1904 (d) 1908 Answer-

Question 50

Which type of tournament is best if there are a large number of teams participating?

(a) Round robin (b) Challenge (c) Combination (d) Elimination Answer-

(d) Elimination

Question 51

League – Cum – Knock Out is a part of:

(a) Knock Out tournament. (b) Round Robin tournament. (c) Combination tournament. (d) Consolation tournament. Answer-

(c) Combination tournament.

Question 52  ISC Physical Education Semester-1 Specimen

Which of the following pairs is incorrect?

(a) AIFF – All India football federation (b) AITA – All India tennis Association (c) BAI – Badminton Association of India (d) HI – India Hockey Answer-

(d) HI – India Hockey

Question 53

Which of the following is not a correct statement about intramurals?

(a) They are recreational sports. (b) They provide opportunities for every individual. (c) They are played inside the campus. (d) They are the competitions with other schools. Answer-

(d) They are the competitions with other schools.

Question 54

Which of the following is not a merit of tournament?

(a) Tournament tests your skills. (b) In team games, all players do not get equal chance. (c) Tournament helps one to overcome fears. (d) Culture exchange. Answer-

(b) In team games, all players do not get equal chance.

Question 55

(I) The team in upper half would be n/2 (II) The teams in lower half would be n/2 With reference to the above formulae, which is the formula for the Single Knock – Out for even number of teams?

(a) Only I (b) Only II (c) Both (I) and (II) (d) None of the two. Answer-

Question 56

Which one of the following is not a type of tournament?

(a) Olympic tournaments (b) Combination tournament (c) Knock-out (d) Round robin tournament Answer-

(a) Olympic tournaments

Question 57

____________ activities are organised amongst the students outside the walls of an institution.

(a) Recreational (b) Amateurs (c) Extramural (d) Intramural Answer-

(c) Extramural

Question 58

Which statement is incorrect about professional sports persons?

(a) They play for money. (b) They play out of compulsion. (c) They train full time. (d) They view sports as a leisure activity. Answer-

(d) They view sports as a leisure activity.

Question 59

Which of the following is not a major international tournament?

(a) National games (b) Asian games (c) Olympic games (d) FIH World cup Answer-

(a) National games

Question 60

Which of the following trophies is not related to cricket?

(a) Ranji Trophy (b) B. C. Roy Trophy (c) Irani Trophy (d) Vijay Hazare Trophy Answer-

(b) B. C. Roy Trophy

Question 61

What is the exact duration of Olympic games?

(a) 18 days (b) 17 days (c) 15 days (d) 16 days Answer-

(b) 17 days

Question 62

Which of the following game is popularly played in the Commonwealth countries?

(a) Hockey (b) Horse polo (c) Cricket (d) Lawn Tennis Answer-

Question 63

The other name of League Tournament is:

(a) Round Robin Tournament (b) Challenge Tournament (c) Knock Out Tournament (d) Combination Tournament Answer-

(a) Round Robin Tournament

Question 64  ISC Physical Education Semester-1 Specimen

Which of the given procedures is followed to avoid competing in the initial round?

(a) Bye (b) Seeding (c) Special Seeding (d) Fixture Answer-

(b) Seeding

Question 65

How many byes are given for 21 teams on the knockout basis?

(a) 11 (b) 17 (c) 18 (d) 15 Answer-

Question 66

Which of the following is not an objective of Intramural tournament?

(a) Learning a variety of games & skill. (b) To help in overall development. (c) To achieve high performance. (d) To provide recreation. Answer-

(c) To achieve high performance.

Question 67

Which of the following competitions is organized for the student of a school, within the school boundaries?

(a) Inter – state (b) Intramural (c) Extramural (d) None of these Answer-

(b) Intramural

Question 68

Tournaments help in:

(a) Development of Social qualities. (b) Development of Sports skills. (c) Selection of players. (d) All of the above. Answer-

(d) All of the above.

Question 69  ISC Physical Education Semester-1 Specimen

The objective of a tournament is:

(a) To find out the best team. (b) To provide a source of recreation for the public. (c) To learn new skills. (d) All of the above. Answer-

Question 70

Bye is a privilege given in

(a) Team sports (b) Individual sports (c) Both (a) and (b) (d) None of the above

(a) Team sports

ISC Specimen Paper Semester-1 Solved Class-12

ISC Class-12 Textbook Solutions, Paper, Notes , Syllabus

ISC Board Paper Class-12 Solved Previous Year Question

Please share with your ISC friends if it is helpful

Percentage Class 8 MCQs RS Aggarwal Exe-6C Goyal Brothers ICSE Maths Solutions

Percentage Class 8 MCQs RS Aggarwal Exe-6C Goyal Brothers ICSE Maths Solutions

Percentage Class 8 RS Aggarwal Exe-6B Goyal Brothers ICSE Maths Solutions

Percentage Class 8 RS Aggarwal Exe-6B Goyal Brothers ICSE Maths Solutions

Bonku Babu's Friend Summary Treasure Chest for ICSE Class 10 English

Bonku Babu’s Friend Summary: Treasure Chest for ICSE Class 10 English

2 thoughts on “ISC Physical Education Semester-1 Specimen Paper Solved Class -12”

It was too beneficial for us

thanks and focus on sem-2 seriously

Leave a Comment Cancel reply

This site uses Akismet to reduce spam. Learn how your comment data is processed .

isc physical education solved question paper

  • Skip to main content
  • Skip to secondary menu
  • Skip to primary sidebar
  • Skip to footer

A Plus Topper

Improve your Grades

ISC Specimen Papers for Class 12 Physical Education 2020, 2019, 2018

February 9, 2024 by Veerendra

Download ISC Specimen Papers 2020 Solved for Physical Education Class 12 Notes  and Marking Scheme PDF. Here we have given ISC Physical Education Question Papers 2020 Solved . Students can view or download the Specimen Papers for ISC 2020 with Answers Class 12 Physical Education for their upcoming examination.

These ISC Board Sample Papers are useful to understand the pattern of questions asked in the board exam. Know about the important concepts to be prepared for ISC Class 12 Physical Education board exam and Score More marks.

Board – Indian School Certificate Examinations (CISCE), www.cisce.org Class – Class 12 Subject – Physical Education Year of Examination – 2020, 2019, 2018, 2017.

ISC Class 12 Physical Education Specimen Papers Solved

www.cisce.org ISC Sample Papers for Class 12 Physical Education are part of ISC Specimen Papers Solved for Class 12 Here we have given ISC Class 12 Physical Education Sample Question Papers for Class 12 Physical Education.

2020
2017

Last 10 Years ISC Question Papers Class 12 Physical Education Solved

  • ISC Physical Education Question Paper 2019 Solved
  • ISC Physical Education Question Paper 2018 Solved
  • ISC Physical Education Question Paper 2017 Solved
  • ISC Physical Education Question Paper 2016 Solved
  • ISC Physical Education Question Paper 2015 Solved

The above ISC Model Paper for Class 12 Physical Education is the official sample paper released by ISC Board as per latest syllabus of Class 12 Indian Certificate of Secondary Education, India.

We hope the ISC Specimen Papers for Class 12 Physical Education, help you. If you have any query regarding ISC Class 12 Physical Education Question Papers Solved, drop a comment below and we will get back to you at the earliest.

  • Picture Dictionary
  • English Speech
  • English Slogans
  • English Letter Writing
  • English Essay Writing
  • English Textbook Answers
  • Types of Certificates
  • ICSE Solutions
  • Selina ICSE Solutions
  • ML Aggarwal Solutions
  • HSSLive Plus One
  • HSSLive Plus Two
  • Kerala SSLC
  • Distance Education

New user? Create an account .

Brilliantly

Content & links.

Verified by Sur.ly

Help us customize the learning for you, Become a member today!

Already have an account? Sign in

Olympiad Questions & Sample Papers for Free

Olympiad exams are held with the motive of bringing more knowledge and confidence in the students. Olympiad questions are based on MCQ type questions. The total number of items is 35, that is to be done in 60 minutes for classes I-IV and classes V-XII; 50 questions are there. Olympiad's paper is divided into three to four parts depending upon the subject choice.

Olympiad is taken on two levels-

The initial level is the school level, while the second one is a national level competitive exam. It is a way through which a child's capability and this can also serve the school in a way that they can check on the fields that require more attention. It also enhances the reasoning and problem-solving skills of the students.

This exam is only meant for students of class I-XII. We provide sample papers and Olympiad questions to reduce the pressure on the students. Solving the sample papers thoroughly is one way of preparing for such competitive exams. It is essential to study the course thoroughly and then solve as many sample papers as we can. More practice will bring better results for the examination. Previous year question papers and Olympiad sample papers are also available to give you an idea of the paper pattern.

Olympiad Questions By Subject

https://www.practice-olympiad.com site provides free sample papers to help your child prepare better for the Olympiad Exams.

We provide Sample Papers for SOF, ASSET, EduHeal Foundation, MacMillan ICAS/IAIS, SilverZone and Unified Council Olympiad Exams.

1. Math Olympiad Questions and Sample Papers

Maths Olympiad is an examination curated with advanced level mathematics to work on a student's current potential. In such tests, the students are required to solve the given questions. Based on the problem solving, the student is rewarded with marks and accolades. To support in this particular situation, several coaching centres and websites are there that provide the necessary support.

We offer an ample amount of Math Olympiad Questions and Maths Olympiad Sample Papers for the student to be ready and practice their way to perfection. We also provide the international level of maths Olympiad questions. For providing extra assistance to prepare for the Olympiads, there is International Maths Olympiad Questions .

2. Science Olympiad Questions and Sample Papers

Science Olympiads are designed with updated questions from every science field to test the science of a student at an advanced level. It can be an excellent way to examine knowledge in all sectors of science at the same time. The general format of an Olympiad exam is that the student must solve the given questions within an amount of specified time. Multiple efforts are made to introduce Olympiads for different age groups nationally or internationally. One of the official brands that are bringing together the Olympiad culture in our country, especially in science, is the NSO.

We have specially made available National Science Olympiad Question Papers, NSO Sample Papers , and NSO Question Papers to help the students prepare. This site will ensure that you are provided with all the advanced levels of various materials and assistance in solving them.

3. English Olympiad Questions and Sample Papers

Olympiads help students in various ways. Olympiads consist of multiple-choice questions, and students are required to select the answer they think is correct. Students are also required to carry their pen and pencils.

English Olympiads specifically will help you improve your grip over the language and even help you with your grammar and vocabulary.

4. Hindi Olympiad Questions and Sample Papers

Hindi Olympiad is an exam that is commonly given by the students during their school time, and this is an important exam to test the Hindi abilities of the student. A student is allotted a set of questions, and they have to cover it in a specified period. You will find Hindi Olympiad Questions and sample papers at this site that will help the student to build a strong base in the Hindi language.

5. General Knowledge Olympiad Questions and Sample Papers

GK Olympiad Questions organized by International General Knowledge Olympiad are the most prominent General Knowledge papers. It is held across the globe and in India, which analyses your child's brain's strengths and weaknesses and lets you recognize which position you have to work on for your child's betterment.

IGKO Sample Papers are provided every year for all primary and secondary classes, starting from 1st to 12th.

6. Cyber Olympiad Questions and Sample Papers

National Cyber Olympiad is a school level competitive exam that tests students' skills in the field of cyber skills. This paper is majorly based on logical as well as computer skills. The syllabus for Computer Olympiad Questions is the one that is there in the school curriculum for classes I-X. It builds confidence amongst students and improves their performance by looking at the students' national average performance.

The paper is in three parts, which are as follows:

  • General Knowledge

isc physical education solved question paper

International Physics Olympiad (IPhO) Past Papers Questions

Physics Olympiad (IPhO) Past papers Questions

physics olympiad past papers questions

International Physics Olympiad (IPhO) is the Toughest Physics Competition for high School students, which held every year in the month of June – July in a different country. It is also known as the World championship Physics Competition.

The main aim of IPhO Contest is to test the highest level of knowledge in Physics, critical thinking, problem solving, right practices of presentation and analysis, and hands-on skills in theoretical and experimental physics.

Here, High school Students or Physics Olympiad candidates will get all the guidance, Notes and the Past papers of IPhO, that will help you to understand about the level of this test and to achieve High score in this contest.

PHYSICS OLYMPIAD PAST QUESTIONS

We are providing the Past year papers Questions & Solutions of Physics Olympiad (IPhO) from the basic to advance. You could increase your concept up to the high level if you will be able to solve all previous questions. Good Luck!

(Planetary Physics)
T2 (Electrostatic Lens)
T3 (Particles and Waves)

(Non-Ideal Capacitors)
E2 (Light emitting Diodes)
(Zero-Length Springs and Slinky Coils)
T2 (Physics of Microwave oven)
T3 (Thermoacoustic engine)

(Optical Measurements)
E2 (Wiedemann-Franz Law)
(LIGO)
T2 (Where Is Neutrino)
T3 (Physics of Live System)

(Paper Transistor)
E2 (Viscoelasticity of Polymer Thread)
(Dark Matter)
T2 (Earthquake Volcano Tsunami)
T3 (Cosmic Inflation)

(Salt Solution)
E2 (Earthquake & Volcanic Sensing)
(Two Problems In Mechanics)
T2 (Nonlinear Dynamics In EI-circuits)
T3 (Large Hadron Collider)

(Particles From Sun)
T2 (Extremum Principle)
T3 (Design of Nuclear Reactor)

(Diffraction Due to Helical Structure)
E2 (Surface Tension Waves on Water)


(Three Problems)
T2 (Van der Waals)
T3 (Gas Discharge)

(See Invisible)
(Maribo Meteorite)
T2 (Plasmonic Steam Generator)
T3 (Greenland Ice)

(Speed of Light)
E2 (Solar cells)
(Focus on Sketches)
T2 (Kelvin Water Dropper)
T3 (Protostar Formation)

(Magnetic Permeability of Water)
E2 (Non-linear Black box)
(Three-body Problem & LISA)
T2 (Electrified Soap Bubble)
T3 (Scattering of Ion)

(Electrical Blackbox)
E2 (Mechnical Blackbox)
(Image of Charge In Metallic Object)
T2 (Chimney Physics)
T3 (Model of Atomic Nucleus)

(Earth-Moon System)
T2 (Laser Cooling & Optical Molasses)
T3 (Star Size)

(Wavelength of Diode Laser)
E2 (Birefringence of MICA)
(Rice Pounding Mortar)
T2 (Cherenkov Light)
T3 (Atmosphere)

(Solidification of Crystalline Substance)
E2 (Efficiency of Solar Cell)
(Black Hole)
T2 (Air Bags of Automobiles)
T3 (Photometry of Binary Stars)

(Semiconductor Thin Films)
(Neutron Interferometer)
T2 (Rod In Motion)
T3 (Digital Camera, Hard-boiled Egg, Lightning, Capillary Vessels)

(Michelson Interferometer, Thin-film Interference, Total Internal Reflection, Bragg Reflection)
(ILL Fated Satellite)
T2 (Measurements of Electrical Quantities)
T3 (Neutrons In Gravity)

(Light of Incandescent Lamp)


(Ping-pong Resistor)
T2 (Rising Balloon)
T3 (Atomic Probe Microscope)

(Mechanical Black box)
(Swing with Falling weight)
T2 (Piezoelectric Crystal Resonator)
T3 (Neutrino Mass & Neutron Decay)

(Laser Diode, Nematic Liquid Crystal)
(Ground-Penetrating Radar)
T2 (Predator & Prey)
T3 (Vehicle Moving on Inclined Road)

(Electrolysis)
E2 (Optical Black Box)
T1
T2
T3

(Rotating Liquid)



(Absorption of Radiation in Gas)
T2 (Magnetic Field)
T3 (Space Probe To Jupiter)

(Torsion Pendulum)
(Hexagonal Prism)
T2 (Water Under Ice Cap)
T3 (Faster Than Light)

(Magnetic Shielding & Flux Linkage)
(Scalings)
T2 (Nuclear Masses & Stablity)
T3 (Solar Powered Aircraft)

(Five problems)
T2 (Electron & Cylinder)
T3 (Moon & tides)

(Physical Pendulum)
(Gravitational Red Shift)
T2 (Sound Propagation)
T3 (Cylindrical Buoy)

(Terminal velocity)
E2 (Diffraction & Scattering of laser light)
(Relativistic Particle)
T2 (Superconducting Magnet)
T3 (Collision of Discs)

(Atmospheric Electricity)
T2 (Laser Forces)
T3 (Electron Beam)

(Rotating Satellite)
T2 (Linear Molecule)
T3 (Satelite in Sunshine)

(Electric Breakdown of Air)
Experimental (Friction on Impact)
T2 (Relativistic Square)
T3 (Cooling Atom By Laser)
E1 (Black Box)
Experimental (X-ray Diffraction)
T2 (Magnetosphere of Earth)
T3 (Rotating Neutron Star)
E1 (Light-Emitting & Photo Diode)
E1 (Magnetic Field Strength)


Experimental (Boiling Liquids)
T2 (Rotating Masses)
T3 (Electron Microscope)
E1 (piezoelectric discs)
Experimental (Spectroscopy of Particle Velocities)
T2 (Maxwell’s Wheel)
T3 (Recombination of Ions)
E1 (Polarized Light)
E2 (Electron Tube)
Experimental (Ascending Moist Air)
T2 (Electrons In Magnetic field)
T3 (Infinite LC-grid)
E1-E2 (Refractive Indices)
(Interference)
T2 (Seismic waves)
T3 (Masses & Springs)

(Pendant Water Droplet)
E2 (Microcomputer)
Experimental (Radio Amateur)
T2 (Hall Effect)
T3 (Launching Space Probe out of Solar System)
E1 (Brass Disk)
Experimental (Transparent Plate)
T2 (Seiching)
T3 (Electronic Frequency Filter)
E1 (Sine Wave Generator)
(Mechanics – Jumping Particle)
T2 (Electricity – Oscillation)
T3 (Optics – Prisms)
T4 (Atomics – Compton Scattering)
T5 (IPhO’s LOGO)

Experimental (Fluorescent lamp)
T2 (Oscillating Coat hanger)
T3 (Hot air Balloon)
T4 (Lens Experiment)
T5 (Motion of Rolling Cylinder)


(Rubber Cord)
Experimental (Space Rocket Moving Around Moon)
T2 (Brass weights Used to Weigh Aluminum)
T3 (Optical Location of Moon)
E1 (Rocket Moving along Circular Orbit of Moon)
E2 (Black Box)




Experimental (Hollow Sphere)
T2 (Cylinder & Piston)
T3 (Air Bubble in Glass Sphere)
E1 (Thermal Properties of Crystal Material)
Experimental (Rotating Rod)
T2 (Thick lens)
T3 (Ions In Magnetic Field)
E1 (Semiconductor Element)
Experimental (Hydrogen Atom in Ground State)
T2 (Transparent Plate)
T3 (Energy Source)
E1 (Black Box)
Experimental (Mechanics – 3 Cylinders)
T2 (Molecular Physics – 2 Cylinder)
T3 (Electricity – Capacitor)
E1 (optics – Thin-Lens Plane-Convex)
E2 (Mechanics – 2 Cylindrical Bodies)
Experimental (Triangular Prism)
T2 (Glass Tube Contain Hydrogen)
T3 (EMF)
E1 (Spherical aquarium)

Experimental (Bar Move Frictionless)
T2 (Natrium Chloride Crystal)
T3 (Thin-walled Metal Sphere)
E1 (Spherical Mirror In Telescope)
Experimental (Mechanical system of Cart)
T2 (Copper Calorimeter)
T3 (Charged Ball)
E1 (Electromagnetic Radiation)
Experimental (Inclined Plane)
T2 (Toluene in Glass)
T3 (Light Rays)
E1 (Black Boxes)
Experimental (Ball)
T2 (Resistors)
T3 (Balls)
E1 (Closed Vessel)
E2 (Specific Heat of Petroleum)

Other Exam Papers

Related Posts

top 200 universities of the world

Top 200 Universities Of The World

biology class 12 neet study notes

Class 12 Biology (Chapters 1-16) Handwritten Notes PDF

thulium

Thulium Tm (Element 69) of Periodic Table

Leave a comment cancel reply.

Your email address will not be published. Required fields are marked *

  • Prime Notes
  • Notes & Past Papers
  • Periodic Table
  • Mechanical Engineering
  • Electrical Engineering
  • Civil Engineering
  • Computer Science (CSE/IT)
  • Modern English
  • Words & Expressions
  • Famous Quotes
  • Religious Quotes
  • Curious Science
  • Why Questions
  • Organizations

IMAGES

  1. ISC Class 12th Physical Education SOLVED Specimen Paper 2023

    isc physical education solved question paper

  2. Physical Education (Theory) 2021-2022 ISC (Commerce) Class 12 Set 1

    isc physical education solved question paper

  3. ISC Class 12th Physical Education SOLVED Specimen Paper 2023

    isc physical education solved question paper

  4. ISC Physical Education Question Paper 2017 Solved for Class 12

    isc physical education solved question paper

  5. ISC Class 12 Physical Education Previous Year Question Papers Download

    isc physical education solved question paper

  6. ISC Question Papers 2013 for Class 12

    isc physical education solved question paper

COMMENTS

  1. ISC Class 12 Physical Education Previous Year Question Papers Solved

    These ISC 12th Physical Education Previous Year Question Papers Solved with Answers are useful to understand the pattern of questions asked in the board exam. Know about the important concepts to be prepared for ISC Class 12 Board Exam and Score More marks. Board - Indian Certificate of Secondary Education (CISCE), www.cisce.org. Class ...

  2. ISC Physical Education Previous Year Question Papers (Solved) PDF (2024

    One of the most important study materials for all the students is ISC Physical Education previous year question papers as they give a brief idea to all the students about the pattern of the examination, marking scheme, weightage per topic etc. Solving the Physical Education previous year question papers of Class 12 regularly will boost the confidence of the students and will also help them to ...

  3. ISC Physical Education Question Paper 2018 Solved for Class 12

    ISC Physical Education Previous Year Question Paper 2018 Solved for Class 12 Section-A (40 Marks) Answer any five questions. Question 1. Explain the role of physical education in the development of personality of an individual. [8] Answer: Personality represents those characteristics of the person that account for consistent patterns of behaviour. Broadly, four factors influence […]

  4. ISC Physical Education Specimen Paper 2024 (PDF)

    ISC Class 12 Solutions for Physical Education - While solving the Physical Education book, if you find any question difficult, then you can use the CISCE Board Solutions. ISC Class 12 Physical Education Question Papers - After completing Physical Education curriculum, solve CISCE board Class 12 previous year question papers to get an idea ...

  5. Physical Education (Theory) Official 2022-2023 ISC (Commerce) Class 12

    Previous year Question paper for CISCE Class 12 Physical Education (Theory)-2023 is solved by experts. Solved question papers gives you the chance to check yourself after your mock test. By referring the question paper Solutions for Physical Education (Theory), you can scale your preparation level and work on your weak areas.

  6. ISC Class 12 Physical Education Previous Year Question Papers Download

    The ISC Board class 12 Physical Education comprises two papers, one for theory and other for practical work. Paper 1: Theory is of 70 marks and is three hours long. Candidates will be given 15 ...

  7. ISC Class 12 Physical Education Question Paper 2023 (PDF)

    The ISC Class 12 previous year question paper for Physical Education contain exact questions framed by the Council for the Indian School Certificate Examinations in that session. Therefore by solving the Physical Education paper, you can boost your own exam preparation for upcoming board exams. ISC Class 12 Physical Education Question Paper 2023

  8. ISC Class 12 Physical Education Previous Year Question Papers : Year

    ISC Class 12, includes a syllabus, past year question papers and marking scheme solutions, and the latest specimen papers. On this page, we have given ISC Class 12 Physical Education Question Papers from 2014 to 2024. These files are available absolutely for FREE. No login information required! ISC Class 12 Physical Education Question Papers.

  9. ISC Class 12 Physical Education Previous Year Question Paper: Download

    Follow the simple steps provided below to download the ISC 12th Physical Education Question Paper PDFs from the official website of CISCE: Step 1: Students need to open the official website of CISCE - cisce.org. Step 2: On the homepage, click on "Examinations" from the navigation bar. Step 3: Then click on the link "ISC Examination".

  10. ISC Previous Year Question Papers Solved Class 12

    Here we have given ISC Board Previous Year Question Papers Solved for Class 12. Students can view or download the Last 10 Years ISC Question Papers Pdf for their Class 12 upcoming examination. These ISC 12th Previous Year Question Papers Solved with Answers are useful to understand the pattern of questions asked in the board exam.

  11. ISC Previous Question Papers Solved Class-12 Last 10 Years

    English Literature ISC Class 12 Previous Year Question Papers Solved. 2019 ISC English Literature Question Paper. 2018 to 2006 English Literature (Not in Latest Syllabus 2021) 2005 ISC English Literature Question Paper. 2004 ISC English Literature Question Paper. 2003 ISC English Literature Question Paper.

  12. ISC Class 12 Physical Education Paper 1 Question Paper ...

    Content Curator. ISC Class 12 Physical Education Paper 1 Question Paper 2023 with Answer Key pdf is available for download here. The exam was conducted by Council for the Indian School Certificate Examinations (CISCE) on March 24, 2023 in the Afternoon Session 2 PM to 5 PM. The question paper comprised a total of 14 questions divided among 3 ...

  13. ISC Class 12 Physical Education Sample Paper 2023-24 Solved PDF

    By solving the ISC Class 12 Physical Education Sample Paper 2023-24, you can understand the different types of questions: multiple choice questions, match the following, and subjective questions. Through this, you can learn to approach the same questions while attempting the Class 12 Physical Education exam.

  14. ISC Class 12 Previous Year Question Papers: Subject-wise Analysis

    Students can directly download the ISC previous year papers for class 12 for the academic session 2024-25 from the CISCE's official website- cisce.org and From ISC PYQs Class 12. CISCE every year uploads the past year papers so that students can curate a plan accordingly to tackle all types of questions. 2.

  15. ISC Physical Education Semester-2 Solved Specimen Paper 2022 ...

    January 5, 2022 by PANDEY TUTORIAL. ISC Physical Education Semester-2 Solved Specimen / Model / Sample Paper 2022 Class-12 for practice. Step by step solutions of ISC Class-12 specimen model sample paper. During solutions of semester-2 Physical Education specimen paper we explain with figure , graph, table whenever necessary so that student can ...

  16. ISC Physical Education Specimen Paper 2024-25 (PDF)

    ISC Board Class 12 Physical Education Sample Paper 2024-25: If you are also a CISCE Board student and looking for the latest sample paper on Physical Education, then don't worry as we got you ...

  17. ISC Board Class 12 Question Papers with Solutions

    Students can download ISC Class 12 Questions Papers starting from the year 2017-2020 from this article. Alternatively, students can download the previous year's question papers from the official website. Follow the below instructions: 1st Step: Log on to the official website cisce.org.

  18. ISC Physical Education Semester-1 Specimen Paper Solved Class -12

    Question 69 ISC Physical Education Semester-1 Specimen. The objective of a tournament is: (a) To find out the best team. (b) To provide a source of recreation for the public. (c) To learn new skills. (d) All of the above. Answer-. (d) All of the above.

  19. ISC Specimen Papers for Class 12 Physical Education 2020, 2019, 2018

    Download ISC Specimen Papers 2020 Solved for Physical Education Class 12 Notes and Marking Scheme PDF. Here we have given ISC Physical Education Question Papers 2020 Solved. Students can view or download the Specimen Papers for ISC 2020 with Answers Class 12 Physical Education for their upcoming examination. These ISC Board Sample Papers are useful to […]

  20. Olympiad Questions & Sample Papers for Free

    6. Cyber Olympiad Questions and Sample Papers. National Cyber Olympiad is a school level competitive exam that tests students' skills in the field of cyber skills. This paper is majorly based on logical as well as computer skills. The syllabus for Computer Olympiad Questions is the one that is there in the school curriculum for classes I-X. It ...

  21. International Physics Olympiad (IPhO) Past Papers Questions

    International Physics Olympiad (IPhO) is the Toughest Physics Competition for high School students, which held every year in the month of June - July in a different country. It is also known as the World championship Physics Competition. The main aim of IPhO Contest is to test the highest level of knowledge in Physics, critical thinking ...

  22. PDF Moscow School Olympiad in Physics

    Moscow Physics Olympiad 2021 11th Final Round 2 A. It is known that the circuit parameters satisfy the relation L/R ˝ ˝. Determine the charge

  23. PDF Highlight of the 10 International Workshop on Microwave Discharges

    The business meeting of ISC was held on 06.09.2018. The planned rotation of ISC member was realized and Prof M. Nagatsu was replaced by Prof. H. Toyoda as a representative of Japan. Taking into account the important contribution of Prof. M. Nagatsu in the success of the Workshop, he was elected the Honorary Member of ISC.